You are on page 1of 43

STEP 3

1. Fisiologi dari nifas dan perubahan2 pada sistem?


Uterus
The pregnant term uterus (not including baby, placenta, fluids, etc) weighs approximately
1000 g. In the 6 weeks following delivery, the uterus recedes to a weight of 50-100 g.

Immediately postpartum, the uterine fundus is palpable at or near the level of the maternal
umbilicus. Thereafter, most of the reduction in size and weight occurs in the first 2 weeks, at
which time the uterus has shrunk enough to return to the true pelvis. Over the next several

weeks, the uterus slowly returns to its nonpregnant state, although the overall uterine size
remains larger than prior to gestation.

The endometrial lining rapidly regenerates, so that by the seventh day endometrial glands are
already evident. By the 16th day, the endometrium is restored throughout the uterus, except
at the placental site.

The placental site undergoes a series of changes in the postpartum period. Immediately after
delivery, the contractions of the arterial smooth muscle and compression of the vessels by
contraction of the myometrium ("physiologic ligatures") result in hemostasis. The size of the
placental bed decreases by half, and the changes in the placental bed result in the quantity
and quality of the lochia that is experienced.

Immediately after delivery, a large amount of red blood flows from the uterus until the
contraction phase occurs. Thereafter, the volume of vaginal discharge (lochia) rapidly
decreases. The duration of this discharge, known as lochia rubra, is variable. The red
discharge progressively changes to brownish red, with a more watery consistency (lochia
serosa). Over a period of weeks, the discharge continues to decrease in amount and color
and eventually changes to yellow (lochia alba). [1] The period of time the lochia can last varies,
although it averages approximately 5 weeks.[2]
The amount of flow and color of the lochia can vary considerably. Fifteen percent of women
have continue to have lochia 6 weeks or more postpartum. Often, women experience an
increase in the amount of bleeding at 7-14 days secondary to the sloughing of the eschar on
the placental site. This is the classic time for delayed postpartum hemorrhages to occur.

Cervix
The cervix also begins to rapidly revert to a nonpregnant state, but it never returns to the
nulliparous state. By the end of the first week, the external os closes such that a finger cannot
be easily introduced.

Vagina
The vagina also regresses but it does not completely return to its prepregnant size.
Resolution of the increased vascularity and edema occurs by 3 weeks, and the rugae of the
vagina begin to reappear in women who are not breastfeeding. At this time, the vaginal
epithelium appears atrophic on smear. This is restored by weeks 6-10; however, it is further
delayed in breastfeeding mothers because of persistently decreased estrogen levels.

Perineum
The perineum has been stretched and traumatized, and sometimes torn or cut, during the
process of labor and delivery. The swollen and engorged vulva rapidly resolves within 1-2
weeks. Most of the muscle tone is regained by 6 weeks, with more improvement over the
following few months. The muscle tone may or may not return to normal, depending on the
extent of injury to muscle, nerve, and connecting tissues.

Abdominal wall
The abdominal wall remains soft and poorly toned for many weeks. The return to a
prepregnant state depends greatly on maternal exercise.

Ovaries
The resumption of normal function by the ovaries is highly variable and is greatly influenced
by breastfeeding the infant. The woman who breastfeeds her infant has a longer period of
amenorrhea and anovulation than the mother who chooses to bottle-feed. The mother who
does not breastfeed may ovulate as early as 27 days after delivery. Most women have a
menstrual period by 12 weeks; the mean time to first menses is 7-9 weeks.

In the breastfeeding woman, the resumption of menses is highly variable and depends on a
number of factors, including how much and how often the baby is fed and whether the baby's
food is supplemented with formula. The delay in the return to normal ovarian function in the
lactating mother is caused by the suppression of ovulation due to the elevation in prolactin.
Half to three fourths of women who breastfeed return to periods within 36 weeks of delivery.

Breasts
The changes to the breasts that prepare the body for breastfeeding occur throughout
pregnancy. If delivery ensues, lactation can be established as early as 16 weeks' gestation.
Lactogenesis is initially triggered by the delivery of the placenta, which results in falling levels
of estrogen and progesterone, with the continued presence of prolactin. If the mother is not
breastfeeding, the prolactin levels decrease and return to normal within 2-3 weeks.

The colostrum is the liquid that is initially released by the breasts during the first 2-4 days after
delivery. High in protein content, this liquid is protective for the newborn. The colostrum, which
the baby receives in the first few days postpartum, is already present in the breasts, and
suckling by the newborn triggers its release. The process, which begins as an endocrine
process, switches to an autocrine process; the removal of milk from the breast stimulates
more milk production. Over the first 7 days, the milk matures and contains all necessary
nutrients in the neonatal period. The milk continues to change throughout the period of
breastfeeding to meet the changing demands of the baby.

http://emedicine.medscape.com/article/260187-overview#aw2aab6b4
2. Macam2 lokea?
There are three stages of lochia:
Lochia rubra -- the first three to five days of postpartum vaginal discharge.
Lochia rubra appears very bright red and contains a large amount of red blood
cells.
Lochia serosa -- occurs until about the tenth day after childbirth. Lochia serosa
is thinner than lochia rubra and is brownish or pink.
Lochia alba -- occurs next, lasting up to six weeks after childbirth. Lochia alba is
white or yellowish-white.
All three stages of lochia have an odor similar to that of normal menstrual flow. If
you experience an offensive or abnormal odor during lochia, contact your
physician for evaluation.

http://pms.about.com/od/glossary/g/lochia.htm

Lochia Flow. One of the most unique capabilities of the uterus is its ability
to rid itself of the debris remaining after delivery. This process is known as
lochia flow. This is the vaginal discharge during the puerperium consisting
of blood, tissue, and mucous. It may last up to six weeks after delivery. It is
important for the nurse, as well as the patient, to be concerned with the
following facts about lochia flow:

(1) Types of lochia (in order of occurrence).

(a) Lochia rubra-a red, distinctly blood-tinged vaginal flow that follows
delivery. It lasts from two to four days after delivery.

(b) Lochia serosa-a serous, pinkish brown, watery vaginal discharge that
follows lochia rubra. It lasts until about the 10th day after delivery.

(c) Lochia alba-a thin, yellowish to white, vaginal discharge that follows
lochia serosa on about the 10th post delivery day. It may last from the end
of the third to the sixth post delivery week.

(2) Lochia with a foul-smell or a green-tinge may indicate infection.

(3) Lochia clots whereas normal menstrual flow does not.

(4) Normal lochia flow should stop within three to four weeks postpartum.

(5) An increase in lochia flow may indicate a retained placenta or a patient


who is not getting enough rest.

(6) Lochia flow is slightly heavier after breast-feeding, which is due to the
release of oxytocin. Oxytocin causes the uterus to contract.

http://www.brooksidepress.org/Products/Obstetric_and_Newborn_Care_II/less
on_6_Section_1.htm
3. Mengapa wanita setelah melahirkan 1 minggu oleh dukun muncul perdarahan
dari jalan lahir dan demam ?

Endometritis is the primary cause of postpartum infection. The most common organisms are
divided into 4 groups: aerobic gram-negative bacilli, anaerobic gram-negative bacilli, aerobic
streptococci, and anaerobic gram-positive cocci. Specifically, Escherichia coli, Klebsiella
pneumoniae, and Proteus species are the most frequently identified organisms.

Endometritis occurring on postpartum day 1 or 2 most frequently is caused by group A


streptococci. If the infection develops on day 3 or 4, the causative organism is frequently
enteric bacteria, most commonly E coli, or anaerobic bacteria. Endometritis that develops
more than 7 days after delivery is most frequently caused by Chlamydia
trachomatis. Endometritis following cesarean delivery is most frequently caused by anaerobic
gram-negative bacilli, specifically Bacteroidesspecies.

Known risk factors for endometritis include cesarean delivery, young age, low socioeconomic
status, prolonged labor, prolonged rupture of membranes, multiple vaginal examinations,
placement of an intrauterine catheter, preexisting infection or colonization of the lower genital
tract, twin delivery, and manual removal of the placenta. It has also been shown that manual
removal of the placenta at cesarean delivery increases the incidence of endometritis.

A patient may report any of the following symptoms: fever, chills, lower abdominal pain,
malodorous lochia, increased vaginal bleeding, anorexia, and malaise.

Physical
A focused physical examination is important and should include vital signs, an examination of
the respiratory system, breasts, abdomen, perineum, and lower extremities. A patient with
endometritis typically has a fever of 38C, tachycardia, and fundal tenderness. Some patients
may develop mucopurulent vaginal discharge, whereas others have scant and odorless
discharge.

http://emedicine.medscape.com/article/260187-overview#aw2aab6b5

Local spread of colonized bacteria is the most common etiology for postpartum
infection following vaginal delivery. Endometritis is the most common infection in the
postpartum period. Other postpartum infections include (1) postsurgical wound
infections, (2) perineal cellulitis, (3) mastitis, (4) respiratory complications from
anesthesia, (5) retained products of conception, (6) urinary tract infections (UTIs),
and (7) septic pelvic phlebitis. Wound infection is more common with cesarean
delivery. (A review study by Haas et al indicated that cleansing the vagina with a
povidone-iodine solution immediately prior to cesarean delivery decreases the risk for
postoperative endometritis.[4] )

http://emedicine.medscape.com/article/796892-clinical

Causes
Causes and risk factors may include the following:

Endometritis
o Route of delivery is the single most important factor in the
development of endometritis.[8]
o The risk of endometritis increases dramatically after cesarean delivery.
[8, 9]

o However, there is some evidence that hospital readmission for


management of postpartum endometritis occurs more often in those who
delivered vaginally.[9]
o Other risk factors include prolonged rupture of membranes, prolonged
use of internal fetal monitoring, anemia, and lower socioeconomic status.[8]
o Perioperative antibiotics have greatly decreased the incidence of
endometritis.[8]
o In most cases of endometritis, the bacteria responsible are those that
normally reside in the bowel, vagina, perineum, and cervix.
o The uterine cavity is usually sterile until the rupture of the amniotic
sac. As a consequence of labor, delivery, and associated manipulations,
anaerobic and aerobic bacteria can contaminate the uterus.
Wound infections
o Most often, the etiologic organisms associated with perineal cellulitis
and episiotomy site infections are Staphylococcus or Streptococcus species and
gram-negative organisms, as in endometritis.
o Vaginal secretions contain as many as 10 billion organisms per gram
of fluid. Yet, infections develop in only 1% of patients who had vaginal tears or
who underwent episiotomies.
o Those who underwent cesarean delivery have a higher readmission
rate for wound infection and complications than those who delivered vaginally.[10]
Genital tract infections
o Increased risk related to the duration of labor (ie prolonged labor
increases risk of infection), use of internal monitoring devices, and number of
vaginal examinations.[11]
o Genital tract infections are generally polymicrobial.
o Gram-positive cocci and Bacteroides and Clostridium species are the
predominant anaerobic organisms involved. Escherichia coli and gram-positive
cocci are commonly involved aerobes.
Mastitis
o The most common organism reported in mastitis is Staphylococcus
aureus.
o The organism usually comes from the breastfeeding infant's mouth or
throat.
o Thrombosis
o Numerous factors cause pregnant and postpartum women to be more
susceptible to thrombosis. Pregnancy is known to induce a hypercoagulable state
secondary to increased levels of clotting factors. Also, venous stasis occurs in the
pelvic veins during pregnancy.
o Although relatively rare, septic pelvic thrombosis is occasionally
observed in the postpartum patient, who might have fever.
Urinary tract infections
o Bacteria most frequently found in UTIs are normal bowel flora,
includingE coli and Klebsiella,Proteus, and Enterobacter species.
o Any form of invasive manipulation of the urethra (eg, Foley
catheterization) increases the likelihood of a UTI.
General risk factors
o History of cesarean delivery
o Premature rupture of membranes
o Frequent cervical examination (Sterile gloves should be used in
examinations. Other than a history of cesarean delivery, this risk factor is most
important in postpartum infection.)
o Internal fetal monitoring
o Preexisting pelvic infection including bacterial vaginosis
o Diabetes
o Nutritional status
o Obesity
In the aforementioned study by Bauer et al, of approximately 45 million hospitalizations for
delivery between 1998 and 2008, medical conditions that were found to be independently
associated with severe sepsis included congestive heart failure, chronic kidney disease,
chronic liver disease, and systemic lupus erythematosus. An association with rescue cerclage
was also found

4. Apa tindakan awal yang dilakukan oleh dokter untuk menghentikan perdarahn?

Medical Therapy
The treatment of patients with PPH has 2 major components: (1) resuscitation and
management of obstetric hemorrhage and, possibly, hypovolemic shock and (2) identification
and management of the underlying cause(s) of the hemorrhage. For the purpose of
discussion, these components are discussed separately; however, remember that successful
management of PPH requires that both components be simultaneously and systematically
addressed.

Management of obstetric hemorrhage


Patients with PPH require aggressive measures to restore and maintain the circulating blood
volume (and thereby perfusion pressure) to vital structures. All medical units involved in the
care of pregnant women must have a protocol for the management of severe obstetric
hemorrhage.[27] Management of massive obstetric hemorrhage outlines such a protocol for a
pregnant woman in either the antepartum or postpartum period. [28] Implement the protocol in a
manner similar to a cardiac arrest protocol, with the same attention to detail and
documentation. Just as with other advanced life support protocols, conduct periodic reviews
and practice drills.

The diagnosis of PPH is established by observing the amount of bleeding and the patients
clinical status. The amount of blood lost and the patients level of consciousness and vital
signs are continually assessed. Once the diagnosis is made, immediately notify appropriate
staff members. The magnitude and underlying cause of the bleeding to some degree dictate
which specialized personnel are called, but a minimum of 1 obstetrician and 1 anesthetist is
necessary. Skilled midwives or nurses can be indispensable. Notifying blood transfusion
services is essential because the timely availability of blood products is likely to be critical. As
in a cardiac arrest, designate an experienced person to document critical information and
times. Ensure the availability of an operating room. The speed with which PPH occurs,
becomes life-threatening, and can be successfully managed with relatively simple
interventions sometimes makes it difficult to decide when to institute the full protocol.

Fluid resuscitation
Fluid resuscitation of women experiencing obstetric hemorrhage is sometimes overly
conservative. Possible reasons for this include (1) blood loss being generally underestimated
both in volume and rapidity, (2) women initially compensating well for losses because of their
good health and the hypervolemia of pregnancy, (3) concerns that overresuscitation leads to
pulmonary edema, and (4) failure to appreciate the dynamics of fluid shifts in the body.

Immediately commence resuscitation. Raising the legs improves venous return and is
consistent with the positioning used to diagnose and treat the underlying causes of bleeding.
Administer oxygen and obtain intravenous access. All intravenous lines started on the labor
ward for other reasons must be placed with cannulas of sufficient gauge if PPH develops.
Twice as much fluid can be infused through a 14-gauge intravenous line compared with an
18-gauge intravenous line over the same time period.[29] During labor, place at least 1
intravenous line in women at risk for PPH; consider a second line in patients at very high risk.
Perform the initial resuscitation with large volumes of crystalloid solution, either normal saline
(NS) or Lactated Ringers solution (LRS), through peripheral intravenous sites. Central
venous access is not required for the vast majority of patients with PPH, but do not delay
establishing such access if necessary. Draw blood for baseline measurements at this time.
NS is a reasonable solution in the labor ward setting because of its low cost and compatibility
with most drugs and blood transfusions. The risk of hyperchloremic acidosis is very low in the
setting of PPH. If large amounts (>10 L) of crystalloid are being infused, a change to LRS can
be considered.

Dextrose-containing solutions, such as 5% dextrose in water or diluted NS in 5% dextrose in


water, have no role in the management of PPH. Remember that the loss of 1 L of blood
requires replacement with 4-5 L of crystalloid because most of the infused fluid is not retained
in the intravascular space but instead shifts to the interstitial space. This shift, along with
oxytocin use, may result in peripheral edema in the days following PPH. Healthy kidneys
easily excrete this excess fluid. Use wide-open initial infusion rates, with the goal of infusing
the required replacement volume over minutes rather than hours. PPH of up to 1500 mL in a
healthy pregnant woman can usually be managed by crystalloid infusion alone if the cause of
bleeding is arrested. Blood loss in excess of this usually requires the addition of a PRBC
transfusion.

Because a large portion of crystalloid fluid volume is lost to the interstitial space, the use of
colloids in resuscitation has been examined. These solutions are largely retained within the
intravascular space and include albumin, dextran, hydroxyethyl starch, and modified fluid
gelatin. A meta-analysis in the Cochrane Library comparing resuscitation with colloid solutions
versus crystalloid favored the use of crystalloids with respect to mortality. [30, 31]

For albumin or plasma protein fraction compared with NS, 18 trials reported data on mortality
in 641 patients. The pooled relative risk from these trials was 1.52 (95% confidence interval,
1.08-2.13). The NS groups had a 1% mortality rate, versus an 11% mortality rate in the colloid
group.

For dextran compared to NS, 8 trials compared reported data on mortality in 668 patients.
The pooled relative risk was 1.24 (95% confidence interval, 0.94-1.65). Two other recent
meta-analyses on the same topic reached the same conclusions.

Large volumes of colloid solutions (>1000-1500 mL/d) can have an adverse effect on
hemostasis. No colloid solution has been demonstrated to be superior to NS, and, because of
the expense and the risk of adverse effects with colloids, crystalloid is recommended. Given
these findings, the authors recommend against the use of colloid solutions in resuscitation
outside the setting of an RCT.

Blood transfusion
Order blood transfusions if blood loss is ongoing and thought to be in excess of 2000 mL or if
the patients clinical status reflects developing shock despite aggressive resuscitation. Data
from various sources suggest that 1 in 16-40 women experiencing PPH requires a blood
transfusion if active third-stage management is used, whereas approximately 1 in 9 requires a
transfusion if expectant management is used. Newer studies tend to have lower transfusion
rates than older studies.[8]

Whole blood is no longer available in most settings, and, for many reasons, PRBCs are
initially used with other blood components and given only if indicated. Most medical units
have access to uncrossmatched O-type Rh-negative PRBCs for catastrophic bleeding. In
PPH, uncrossmatched ABO- and Rh-compatible blood is usually available because a blood
group and antibody screen has already been performed. Have full crossmatched blood
available for transfusion within 30 minutes. Clinicians must be aware of the capabilities of
their blood bank regarding timing, type, and amount of blood products available in
emergencies. Good communication with the blood transfusion service is essential, and the
nature of the emergency and the potential amount of blood products required must be
stressed.
The goal is to rapidly transfuse 2-4 U of PRBCs to replace lost oxygen-carrying capacity and
to restore circulating volume. Administer the blood transfusion through a set with an
integrated filter, and use a blood warmer if the infusion rate (>100 mL/min) or the total volume
infused is high. A rapid infusion set with an integrated warmer or a pressure cuff may be used
to increase the infusion rate. PRBCs are very viscous, reducing the infusion rate. This
problem may be overcome by adding 100 mL of NS to each unit. Do not use LRS for this
purpose because the calcium contained in the solution may cause clotting.

The risks of transfusion are well known and are covered elsewhere (eg, seeTransfusion
Reactions or Transfusion and Autotransfusion), but they include infection, transfusion
reaction, and development of atypical antibodies. Several other complications may be noted
in large-volume transfusions. The risk of hypothermia is minimized by the use of blood
warmers. Dilutional coagulopathy may be observed and is discussed below. Hyperkalemia
and acidosis related to the use of stored blood are theoretical risks but are seldom clinically
important if perfusion of vital organs is maintained. Monitor electrolyte and acid-base status if
the situation is ongoing. Hypocalcemia due to citrate intoxication is also seldom observed. [32]

Patients may refuse a transfusion of blood products based on religious or other grounds. A
patient's refusal of blood products must be respected and must not be equated with a desire
for no intervention or be seen as an excuse for suboptimal care. Several options, including
the use of autotransfusion, can be considered for the management of these patients. Ensure
that a care plan is in place (seeTransfusion and Autotransfusion). An article by Hughes et al
reviewed the issues and management options in patients who refuse transfusion. Clinicians
should bear in mind that the refusal may not extend to all related products. Products that use
recombinant technologies such as human erythropoietin and activated factor VIIa are usually
acceptable.[33]

Coagulopathy
Women experiencing PPH do not usually have a preexisting disorder of hemostasis; however,
initial blood work includes a coagulation screen and platelet count. In previously healthy
women, dilutional coagulopathy is not usually observed until approximately 80% of the
original blood volume has been replaced. Regularly monitor hemostatic test results in all
women who require a massive transfusion. If findings are abnormal in conjunction with
ongoing bleeding or oozing from puncture sites, mucous surfaces, or wounds, additional
blood products are required. Infuse fresh frozen plasma (FFP), beginning with 4 U and
following with additional units to normalize the coagulation test findings. Many authorities
recommend the addition of 1 U of FFP for every 5 U of PRBCs for patients who require
continued transfusion.

Thrombocytopenia is likely after 1.5-2 times the blood volume has been replaced. Keep the
platelet count more than 50 X 109/L by using platelet transfusion. Each unit of platelets
increases the platelet count by approximately 10 X 10 9/L. (Platelets are usually given in packs
of 5-6 U.) If bleeding is continuing and the platelet count is less than 50 X 10 9/L, administer
10-12 U initially. If surgical intervention is necessary, maintain the platelet count at more than
80-100 X 109/L. Platelet preparations contain some RBCs, and the administration of anti-D
immunoglobulin (RhoGAM, WinRho) is recommended for Rh-negative women after the crisis
has passed.[34]

If coagulation test results are abnormal from the onset of PPH, strongly consider an
underlying cause (eg, abruptio placenta, HELLP syndrome, fatty liver of pregnancy,
intrauterine fetal demise, amniotic fluid embolus, septicemia, preexisting disorder). Take
specific steps to treat the underlying cause and the hemostatic abnormality.

DIC may also develop if shock has led to marked hypoperfusion of tissues, causing damage
and release of tissue thromboplastins. In such cases, laboratory test results reveal that the D-
dimer levels are elevated and fibrinogen levels are very low, with a prolonged thrombin time.
The management of DIC is identical to that for a patient with dilutional coagulopathy.
Restoration and maintenance of circulating volume along with blood product replacement is
essential.
Cryoprecipitate may be useful along with FFP because of the markedly depressed fibrinogen
levels. Cryoprecipitate provides a more concentrated form of fibrinogen and other clotting
factors (VIII, XIII, von Willebrand factor) and is faster to prepare in the blood bank. It is
commonly given in 6- to 12-U doses and may also be helpful immediately before any surgical
intervention in patients with abnormal coagulation test results. The use of heparin and
antifibrinolytic therapy is not recommended in women with DIC of obstetric origin.

Interest in and experience with recombinant activated factor VIIa (RFVIIa) in massive
hemorrhage situations is growing. This experience has extended to severe postpartum
hemorrhage and results have been encouraging.[35] RFVIIa has been used when conventional
medical management has been unsuccessful and also when varying degrees of surgical
management, up to and including hysterectomy have failed. Therapy is very expensive and
some suggest that use in less severe cases may be potentially harmful. [36] RFVIIa may also be
useful in cases of severe PPH complicated by DIC.[37] Further study is required before
recombinant activated factor VII is put into widespread use.

Seek the advice of a hematologist in cases of massive transfusion or coagulopathy.

Response to resuscitation
Pay close attention to the patients level of consciousness, pulse, blood pressure, and urine
output during the course of the management of massive hemorrhage. A urine output of 30
mL/h or more likely indicates adequate renal perfusion. Closely monitor the CBC count,
coagulation, and blood gas values in addition to acid-base status. Pulse oximetry is useful for
evaluating tissue perfusion and oxygen saturation. Frequent auscultation of the lung fields
helps detect pulmonary edema or the development of adult respiratory distress syndrome. For
patients in critical condition with ongoing bleeding, the placement of a central venous line may
be helpful for resuscitation. Arterial line placement also may aid in monitoring blood pressure
and allowing easy access for blood work. Few patients experiencing PPH require such
invasive monitoring; however, consultation with appropriate specialists and placement in an
intensive care setting are preferred for those who do.

Management of massive obstetric hemorrhage


The following is a plan for managing massive obstetric hemorrhage, adapted from Bonner.
[28]
The word order is a useful mnemonic for remembering the basic outline.

Organization
1. Call experienced staff (including obstetrician and anesthetist).
2. Alert the blood bank and hematologist.
3. Designate a nurse to record vital signs, urine output, and fluids and drugs
administered.
4. Place operating theater on standby.
Resuscitation
1. Administer oxygen by mask.
2. Place 2 large-bore (14-gauge) intravenous lines.
3. Take blood for crossmatch of 6 U PRBCs, and obtain a CBC count,
coagulation screen, urea level, creatinine value, and electrolyte status.
4. Begin immediate rapid fluid replacement with NS or Ringer lactate solution.
5. Transfuse with PRBCs as available and appropriate.
Defective blood coagulation
1. Order coagulation screen (International Normalized Ratio, activated partial
thromboplastin time) if fibrinogen, thrombin time, blood film, and D-dimer results are
abnormal.
2. Give FFP if coagulation test results are abnormal and sites are oozing.
3. Give cryoprecipitate if abnormal coagulation test results are not corrected
with FFP and bleeding continues.
4. Give platelet concentrates if the platelet count is less than 50 X 10 9/L and
bleeding continues.
5. Use cryoprecipitate and platelet concentrates before surgical intervention.
Evaluation of response
1. Monitor pulse, blood pressure, blood gas status, and acid-base status, and
consider monitoring central venous pressure.
2. Measure urine output using an indwelling catheter.
3. Order regular CBC counts and coagulation tests to guide blood component
therapy.
Remedy the cause of bleeding
1. If antepartum, deliver the fetus and placenta.
2. If postpartum, use oxytocin, prostaglandin, or ergonovine.
3. Explore and empty the uterine cavity, and consider uterine packing.
4. Examine the cervix and vagina, ligate any bleeding vessels, and repair
trauma.
5. Ligate the uterine blood supply (ie, uterine, ovarian, and/or internal iliac
arteries).
6. Consider arterial embolization.
7. Consider hysterectomy.
Management of the underlying cause of PPH
Initial assessment
The patients risk factors and the events leading to the diagnosis of PPH may suggest an
underlying etiology, but knowledge that most cases are caused by uterine atony and the need
to be systematic argues for a planned, stepwise approach to assessment and management.
The status of the patient, the severity of the bleeding, and the response to initial management
steps determine if and when the protocol for massive obstetric hemorrhage is instituted.

Uterine atony
Two well-designed trials indicate that oxytocin should be the drug of choice for both
prophylaxis and treatment of post partum hemorrhage caused by uterine atony. [38]

Assess uterine size and tone by placing a hand on the uterine fundus and massaging the
uterus, which serves to express any clots that have accumulated in the uterus or vagina. If the
uterus is found to be boggy and not well contracted, commence vigorous massage and
therapeutic oxytocin. Oxytocin can be administered as a 5-U intravenous bolus, as 20 U in 1 L
of NS intravenously run as fast as possible, or as 10 U intramyometrially with a spinal needle
if no immediate intravenous access is available.

Emptying the bladder may aid in ongoing assessment and facilitate uterine contraction and
subsequent therapeutic maneuvers. Wearing a waterproof gown, elbow-length gloves, and
eye protection is prudent during the management of PPH. Sterile technique is used.

If the uterus remains atonic, commence bimanual massage. A hand is placed on the fundus,
and the second hand is placed anterior to the cervix in the vagina. Prepare the perineum and
vagina. The vaginal hand may be covered in povidone-iodine solution (Proviodine) or a
lubricant to allow it to enter the vagina with less difficulty. Take care to minimize the chance of
causing or worsening trauma in the lower genital tract. Trauma to the vaginal sidewalls and
cervix may be palpated as the hand is gently introduced into the vagina, and blood clots may
be evacuated from the vagina, cervix, and lower uterine segment.

The vaginal hand is placed in the anterior fornix, and the abdominal hand is placed on the
posterior aspect of the fundus. The uterus is raised from the pelvis, pivoted anteriorly, and
compressed between the two hands. The compression expels clots and decreases bleeding.
Massaging the uterus between the hands aids in promoting and sustaining contraction.
Bimanual massage results in a decrease in bleeding, even if the uterus remains relatively
atonic, thus allowing resuscitation a chance to begin to catch up with blood loss.

Use other uterotonic agents if the uterus remains atonic despite oxytocin administration and
bimanual massage. The traditional second-line agent for uterine atony has been ergonovine
(or ergotrate) given as an initial dose of 100 or 125 mcg intravenously or intramyometrially or
200 or 250 mcg intramuscularly. The maximum total dose is 1.25 mg. Hypertension is a
relative contraindication. In some regions, the availability of ergot preparations has become
problematic. Every effort should be made to secure supplies of this inexpensive and useful
agent.
Many authorities now recommend the use of intramuscular carboprost as the second-line
agent when it is available. The recommended dose is 250 mcg intramuscularly or
intramyometrially, not to exceed 2 mg (8 doses). Asthma is a relative contraindication.
Carboprost has been shown to be 80-90% effective in stopping PPH in cases refractory to
oxytocin and ergonovine. Intramuscular administration of these agents is not recommended if
the patient demonstrates evidence of shock because absorption would be compromised.

Misoprostol may also become a valuable agent in the treatment of PPH. One small case
series reported that a dose of 1000 mcg given rectally was effective in causing sustained
uterine contraction in 14 cases refractory to oxytocin, ergonovine, or both. [39, 40] Recent trials
are examining whether the more rapid onset of sublingual/buccal misoprostol will improve its
efficacy in the setting of acute PPH.[41] At this time, however, misoprostol remains a third-line
agent in the management of PPH.[42] The low cost of the drug and its heat stability (does not
require refrigeration) makes it especially appealing for use in the developing world. More trials
are pending.

Winikoff et al examined sublingual (SL) misoprostol for PPH when oxytocin is not feasible to
administer. Oxytocin is considered the standard of care for treating postpartum hemorrhage,
but because of refrigeration requirements and the need for intravenous administration, it is not
always clinically viable, particularly in primitive clinical settings. Active bleeding was controlled
within 20 min for 440 (90%) women administered misoprostol 800 mcg SL (n=488) and 468
(96%) administered oxytocin 40 units IV (n=490) (relative risk [RR], 0.94; 95% confidence
interval [CI], 0.91-0.98). Additional blood loss of 300 mL or greater after treatment occurred for
147 (30%) of women receiving misoprostol and 83 (17%) receiving oxytocin (RR, 1.78; 95%
CI, 1.40-2.26). The authors concluded that in circumstances where it is not feasible to use
oxytocin for postpartum hemorrhage, misoprostol is a suitable alternative. [43]

The investigational agent carbetocin has been compared with oxytocin for prevention of
postpartum hemorrhage. Attilakos et al compared the effectiveness of carbetocin and oxytocin
when given for postpartum hemorrhage after cesarean delivery in a double-blind, randomized
trial. The primary outcome measure was women who required additional pharmacologic
oxytocic interventions. Results showed that significantly more women required additional
oxytocics in the oxytocin group compared with the carbetocin group. [44]

Retained tissue
If the uterus continues to contract poorly or to relax when bimanual compression and
massage are stopped despite the administration of uterotonics, perform manual exploration.
Some authorities advocate earlier exploration; however, this is difficult without general
anesthesia unless the patient is in severe shock or an epidural is already in place. Nitrous
oxide (Entonox) may be useful in facilitating manual exploration if general anesthesia is not
available.

Ensure that resuscitation is well underway by this time, and, if not already started, institute the
massive hemorrhage protocol. If possible, keep the vaginal hand in situ throughout because it
minimizes patient discomfort, the risk of iatrogenic trauma, and, possibly, the risk of
subsequent infection. If the placenta was not delivered before the onset of PPH, an attempt is
now made to deliver it with cord traction and uterine countertraction. Care must be taken
because the risk of uterine inversion is greater if the uterus remains poorly contracted.
Perform manual removal if the placenta is not easily delivered or the cord is avulsed.

Perform manual removal with a level of analgesia that matches the clinical urgency of the
situation. The hand is passed through the cervix and into the lower segment. Care is taken to
minimize the profile of the hand as it enters, keeping the thumb and fingers together in the
shape of a cone in order to avoid damage. Control of the uterine fundus with the other hand is
essential. If the placenta is encountered in the lower segment, it is removed. If the placenta is
not encountered, the placental edge is sought. Once found, the fingers gently develop the
space between the placenta and uterus and shear off the placenta. The placenta is pushed to
the palmar aspect of the hand and wrist, and, once it is entirely separated, the hand is
withdrawn. Do not stop uterotonics while the manual removal is being performed. Restart
bimanual massage, and have an assistant examine the placenta for completeness.
If the placenta has been previously delivered, then exploration of the uterus is still indicated at
this time. The hand is introduced in the same manner, with control of the uterine fundus with
the other hand. Any clots are removed. The cavity is gently explored with attention to any
defects suggestive of uterine rupture. Rupture in the absence of a previous scar is
uncommon. Rupture or dehiscence of a previous lower segment scar does not usually bleed
heavily. The presence of a uterine rupture dictates that a laparotomy be performed.

A partial uterine inversion can be detected as the hand is introduced, just as a complete
uterine inversion would have been detected as the hand was placed in the vagina. If the
condition is encountered, return the uterus to its normal position by pressure on the inverted
fundus from within the uterus. If retained placental tissue is encountered, it is sheared off the
uterine wall and delivered. Adherent placental fragments may be left in situ or removed by
gentle curettage. The risks of curettage include uterine perforation and increased bleeding
caused by laceration of uterine vessels. This may be somewhat minimized by the use of a
large, dull curette. Fragments left in situ may be removed by curettage sometime after the
crisis has passed, although an increased risk of infection probably ensues.

The administration of short-term, broad-spectrum antibiotics following manual removal,


manual exploration, or instrumentation of the uterus in this context is commonly advocated.
Evidence is very limited, but a single small, randomized trial supports the practice. [45]

Immediately resume bimanual massage and compression following exploration and


evacuation of the uterus. Continue infusion of oxytocin, and administer repeat doses of other
uterotonics if the uterus fails to contract and maximal doses have not already been given. The
uterus may contract well, and bleeding abates with massage, followed by uterine relaxation
and increased bleeding when compression and massage are stopped. Prolonged massage at
this point may allow the uterus to contract and retract if it can be kept empty of clots and if
perfusion can be improved with adequate resuscitation. Any period of decreased bleeding
allows fluid and blood component replacement to exceed blood loss and help improve the
patients status.

Surgical management is necessary if the uterus does not remain contracted and bleeding
persists despite all efforts. Packing of the uterus may be an option until the operating room is
ready or if surgery is not an immediately available option. Uterine packing fell into disfavor
during the 1960s as being nonphysiological, concealing ongoing blood loss, and increasing
the risk of infection; however, reports since then have been favorable in very select
circumstances when all previously mentioned maneuvers have failed. [46] The uterus and
vagina must be tightly packed with continuous, layered, 2- or 4-inch gauze under direct
visualization using a speculum and/or retractors or a purpose-built uterine packer. [47] At times,
packing may serve as a definitive treatment. In these cases, the packing is usually removed in
24-48 hours in a setting where recurrent bleeding can be managed if it occurs.

Intrauterine catheters for tamponade of bleeding have also been used. In the past, large bulb
Foley catheters or Sengstaken-Blakemore tubes have been used. [48]More recently, experience
has been gained using catheters specifically designed for postpartum hemorrhage. One such
device is the SOS Bakri tamponade balloon (Bakri, 2001). In low resource settings, condoms
and surgical gloves have been used successfully to control bleeding. [49] Anti-shock garments
are also being evaluated in low resource settings for both the definitive treatment of uterine
atony as well as a method to allow time to bring other treatments to bear [50]

Manual examination helps to exclude a cervical or vaginal laceration, but direct visualization
confirms that bleeding is coming from the uterus and excludes the possibility of missing
trauma to the lower genital tract. If packing is meant to be definitive treatment, then ongoing
assessment of uterine size, blood loss, and patient status must be maintained. Continue
uterotonics and commence broad-spectrum antibiotics. Remove the pack in 24-36 hours in a
setting that allows for appropriate management if bleeding recurs. Packing may also be used
as a temporizing measure before arterial embolization (see Selective arterial embolization).
Isolated reports of successful uterine tamponade with balloon devices have also been
published.[51]

Genital tract trauma


Genital tract trauma is the most likely cause if bleeding persists or is present despite a well-
contracted uterus. Use appropriate analgesia along with good lighting and positioning, which
facilitates excellent exposure. If not already initiated, moving the patient to an operating room
is reasonable at this time. Experienced assistants and an excellent circulating nurse are
essential.

Directly visualize and inspect the cervix with the aid of ring forceps. The anterior lip is
grasped, and the cervix is inspected by using a second ring forceps placed at the 2-oclock
position, followed by progressively "leap-frogging" the forceps ahead of one another until the
entire circumference has been inspected. Small, nonbleeding lacerations of the cervix do not
need to be sutured. Suture any laceration that is bleeding significantly or appears to have the
potential to bleed significantly. Each side of the laceration can be grasped with a ring forceps
back from the torn edge, and gentle traction can be used to aid exposure.

Use an absorbable, continuous interlocking stitch, and use tapered (rather than cutting)
needles for all repairs except for the perineal skin. Ensure that the stitch begins above the
apex of the tear, as with vaginal lacerations and episiotomies. If the apex cannot be
visualized, place the stitch as high as possible and then use it to apply gentle traction to bring
the apex into view. Polyglycolic sutures have largely replaced catgut; however, the latter may
be somewhat less likely to tear the friable tissues of the cervix and vaginal vault and may thus
be useful in repairing lacerations in these areas. The laceration must be observed for
bleeding after the torn edges of the cervix are approximated. The ring forceps can be
replaced and left on for some time if oozing persists.

Lacerations of the vaginal vault must be well visualized and their full extent realized prior to
repair. Lacerations high in the vaginal vault and those extending up from the cervix may
involve the uterus or lead to broad ligament or retroperitoneal hematomas. The proximity of
the ureters to the lateral vaginal fornices, and the base of the bladder to the anterior fornix,
must be kept in mind when repair is undertaken in these areas. Poorly placed stitches can
lead to genitourinary fistulas. An absorbable, continuous interlocking stitch is used. The stitch
must start and finish beyond the apices of the laceration. Great care must be taken because
the tissue is usually very friable. Take a good amount of tissue, and ensure that the needle
reaches the full depth of the tear. Ongoing bleeding and hematoma formation are possible if
small bites are taken.

Again, the laceration must be observed for bleeding after the repair is complete. Pressure or
packing over the repair may achieve hemostasis or allow for better placement of further
hemostatic stitches. Cervical and vaginal vault lacerations that continue to ooze or those that
are associated with hematomas may be amenable to selective arterial embolization (see
Selective arterial embolization).

Traumatic hematomas are rare and may be related to lacerations or may occur in isolation.
They include vulvar and paravaginal hematomas in the lower genital tract and broad ligament
and retroperitoneal hematomas adjacent to the uterus. Patients with lower genital tract
hematomas usually present with intense pain and localized, tender swelling. Broad ligament
hematomas may be palpated as masses adjacent to the uterus. All may result in significant
blood loss that mandates resuscitation.

Lower genital tract hematomas are usually managed by incision and drainage, although
expectant management is acceptable if the lesion is not enlarging. [52] Any bleeding vessels are
tied off, and oozing areas may be oversewn. Place a Foley catheter because urinary retention
can occur because of pain and tissue distortion. Vaginal packing may be useful following
drainage and repair of a paravaginal hematoma. Remove the pack in 24-36 hours.
Embolization may be used in both vaginal and vulvar hematomas that are unresponsive to
surgical management.

Broad ligament and retroperitoneal hematomas are initially managed expectantly if the patient
is stable and the lesions are not expanding.[53] Ultrasound, CT scanning, and MRI all may be
used to assess the size and progress of these hematomas. Selective arterial embolization
may be the treatment of choice if intervention is required in these patients. Use surgical
procedures to evacuate the hematoma, and attempt to tie off any bleeding vessels. Consider
involving a surgeon with extensive experience operating in the retroperitoneal space.
Coagulopathy
If manual exploration has excluded uterine rupture or retained placental fragments, bleeding
from a well-contracted uterus is most commonly due to a defect in hemostasis. A review of the
history and risk factors along with coagulation test results clarifies this diagnosis. Proceed
with blood product replacement as previously described in order to correct abnormalities of
hemostasis. If the coagulation status is normal and bleeding is ongoing despite a well-
contracted uterus, then the possibilities of uterine rupture or an inadequately repaired uterine
incision (if the patient had a cesarean delivery) must be considered. Revisit any repair to the
cervix or vagina before proceeding to surgical management.

Surgical Therapy
Ongoing bleeding secondary to an unresponsive and atonic uterus, a ruptured uterus, or a
large cervical laceration extending into the uterus requires surgical intervention. Laparotomy
for PPH following a vaginal delivery is rare. In a review of emergency peripartum
hysterectomies over a 5-year period in Los Angeles, Calif, the rate was 1 in 1000 deliveries,
but most of these cases began as cesarean deliveries, usually for placenta previa. [54] A study
from Boston, Mass, found a rate of 1.5 in 1000 deliveries with similar risk factors. [55] Canadian
and Irish studies put the rate at 0.4 and 0.3 per 1000 deliveries, respectively.

Adequately resuscitate the patient before surgery. This includes optimizing hemoglobin and
coagulation status as previously described. Fully inform anesthetic and operating room staff
as to the nature of the case. Schedule for a second surgeon to be in attendance, if possible.
As mentioned previously, sustained bimanual compression and massage and uterine packing
may be used to gain time to mount a surgical response. Military antishock trousers provide
the equivalent of an approximately 500- to 1000-mL autotransfusion and potentially gain time
during a resuscitation. Only the leg portion of the trousers are inflated in the setting of PPH.
Direct compression of the aorta may be performed for a short period while the operating room
is prepared.

A recent systematic review examined various techniques used when medical management is
unsuccessful. These included arterial embolization, balloon tamponade, uterine compression
sutures, and iliac artery ligation or uterine devascularization. At present, no evidence
suggests that any one method is more effective for the management of severe PPH.
Randomized controlled trials of the various treatment options may be difficult to perform.
Balloon tamponade is the least invasive and most rapid approach and may thus be the logical
first step.[56]

Laparotomy
The choice between a subumbilical vertical incision and a Pfannenstiel incision for entry into
the abdomen is left to the individual surgeon. Both entries have support, and no strong
evidence indicates that either is superior in this setting. [57] If concern exists regarding pathology
in the upper abdomen or if exposure is thought to be a concern, the vertical incision is
recommended. Broad-spectrum antibiotic coverage is advised.

Upon entry, remove any free blood and inspect the uterus and surrounding tissues for
evidence of rupture or hematoma. If uterine rupture is found, a rapid decision must be made
concerning the viability of repair versus hysterectomy. Bleeding may be reduced in either
instance by grasping bleeding points on the torn edges with clamps. The number of layers
used for any repair is dictated by the thickness of the tissue and the hemostatic response to
suturing. Principles are similar to those of cesarean delivery incision repair. Ensure that
bleeding is stopped and not merely internalized because this would result in ongoing vaginal
bleeding or hematoma formation. Any repair must be carefully observed for hemostasis
before abdominal closure is performed. Uterine exteriorization may improve exposure and
decrease operating time, but great care must be taken to not worsen uterine trauma and to
keep the uterus warm and well perfused to avoid worsening atony.

Hemostasis must be reassessed after the uterus is returned to the abdominal cavity. Consider
placement of a suction drain.
If the uterus is intact upon entry and the bleeding has been caused by atony, then direct
bimanual massage and compression may be performed while systemic uterotonics are
continued. Direct injection of oxytocin, carboprost, and/or ergonovine may be successful in
overcoming atony.

Uterine artery ligation


Uterine artery ligation is a relatively simple procedure and can be highly effective in controlling
bleeding from uterine sources. These arteries provide approximately 90% of uterine blood
flow. The uterus is grasped and tilted to expose the vessels coursing through the broad
ligament immediately adjacent to the uterus. Ideally, place the stitch 2 cm below the level of a
transverse lower uterine incision site. A large atraumatic (round) needle is used with a heavy
absorbable suture. Include almost the full thickness of the myometrium to anchor the stitch
and to ensure that the uterine artery and veins are completely included. The needle is then
passed through an avascular portion of the broad ligament and tied anteriorly. Opening the
broad ligament is unnecessary. Perform bilateral uterine artery ligation. While the uterus may
remain atonic, blanching is usually noted and blood flow is greatly diminished or arrested.

Local oozing may be controlled with direct injection or compression with warm saline packs.
In a series of 265 cases, a 95% success rate was reported using this procedure in PPH
unresponsive to uterotonics in patients who had cesarean births. [58] Another series of 103
cases had a 100% success rate if a stepwise approach was taken. [59] After initial uterine artery
ligation, subsequent stitches were placed 2-3 cm below the initial stitches following bladder
mobilization, and, finally, ovary artery ligation was performed if required. Menstrual flow and
fertility were not adversely affected.

Ovarian artery ligation


The ovarian artery arises directly from the aorta and ultimately anastomoses with the uterine
artery in the region of the uterine aspect of the uteroovarian ligament. Ligation is performed
just inferior to this point in a manner similar to that of uterine artery ligation. The amount of
uterine blood flow supplied by these vessels may increase following uterine artery ligation.
The procedure is easy to perform; however, the potential benefit must be weighed against the
time required to perform the ligations.

Internal iliac (hypogastric) artery ligation


Internal iliac artery ligation can be effective to reduce bleeding from all sources within the
genital tract by reducing the pulse pressure in the pelvic arterial circulation. One study
indicated that pulse pressure was reduced by 77% with unilateral ligation and by 85% with
bilateral ligation.[60] Hypogastric artery ligation is much more difficult to perform, more
commonly associated with damage to nearby structures, and less likely to succeed than
uterine artery ligation. One study reported a success rate of 42%. In patients who undergo
hypogastric artery ligation, uterine artery ligation has usually already failed.

Prerequisites for the procedure include a stable patient, an operator experienced in the
procedure, and a desire to maintain reproductive potential. The retroperitoneal space is
entered by incising the peritoneum between the fallopian tube and the round ligament. The
ureter must be identified and reflected medially with the attached peritoneum. The external
iliac artery is identified on the pelvic sidewall and followed proximally to the bifurcation of the
common iliac artery. The ureter passes over the bifurcation. The internal iliac artery is
identified and followed distally approximately 3-4 cm from its point of origin. The loose areolar
tissue is carefully cleared from the artery. A right-angle clamp is passed beneath the artery at
this point, with great care to avoid damage to the underlying internal iliac vein.

A recommendation is to pass the clamp from lateral to medial in order to minimize the chance
of damage to the adjacent external iliac vessels. Gentle elevation of the artery with a Babcock
clamp facilitates this maneuver.

Ligate the artery with heavy absorbable suture, but do not divide it. Palpate the femoral and
distal pulses before and after the ligation to ensure that the external or common iliac artery
was not inadvertently ligated. If possible, place the ligation distal to the posterior division of
the artery because this decreases the risk of subsequent ischemic buttock pain. Identification
of the posterior division may be difficult, and ligation 3 cm from the internal iliac artery origin
usually ensures that it is not included.

Hysterectomy is required if internal iliac artery ligation is unsuccessful. Patients in whom


internal iliac artery ligation has failed have greater morbidity than those in whom the
procedure has not been attempted. The likelihood of benefit from the procedure must be
balanced against the potential risks. The advent of more effective uterotonic agents, the fact
that most cases of intractable hemorrhage are now related to abnormalities of placentation
that are diagnosed or suggested before delivery, and the option of embolization have
lessened the use of hypogastric artery ligation. The number of surgeons comfortable using
this procedure and the opportunities to teach it are rapidly declining.

Hysterectomy
Hysterectomy is curative for bleeding arising from the uterine, cervical, and vaginal fornices.
The procedure of peripartum hysterectomy is well described in several texts and articles
(eg, Hysterectomy), and the technique differs little from that in nonpregnant patients. [61, 57] While
the organ is more vascular, the tissue planes are often more easily developed. Total
hysterectomy is preferred to subtotal hysterectomy, although the latter may be performed
faster and be effective for bleeding due to uterine atony. Subtotal hysterectomy may not be
effective for controlling bleeding from the lower segment, cervix, or vaginal fornices. Take
every opportunity to become involved when peripartum hysterectomies are performed.

Selective arterial embolization


Angiographic embolization in the management of PPH was first described more than 30 years
ago.[62] As with all of the surgical and most of the medical treatments of PPH, no RCTs
regarding its effectiveness have been conducted. This is likely to remain the case for some
time given the relative rarity of intractable PPH. Several case series suggest that selective
arterial embolization may be useful in situations in which preservation of fertility is desired,
when surgical options have been exhausted, and in managing hematomas. [63] Follow-up of
women undergoing successful embolization for severe intractable PPH reports that women
almost invariably have a return to normal menses and fertility. [64]

The major drawbacks of the procedure are the requirement for 24-hour availability of
radiological expertise and the time required to complete the procedure. Patients must be
stable to be candidates for this procedure. Complications include local hematoma formation at
the insertion site; infection; ischemic phenomena, including uterine necrosis in rare instances;
and contrast-related adverse effects. Currently, most PPH cases requiring hysterectomy are
related to placenta previa. These patients are commonly diagnosed before delivery and are
usually delivered by elective cesarean birth. This planning may allow increased use of
invasive radiological services in the management of such cases.

A retrospective study by Park et al indicated that transcatheter arterial embolization (TAE) is


safe and effective for secondary PPH. In the study, the procedure was clinically successful in
47 of 52 patients (90.4%) being treated for secondary PPH (caused in 23 cases by retained
placenta). Gelatin sponge particles were used in 48 patients, either alone or in combination
with permanent embolic materials (eg, microcoils, N-butyl cyanoacrylate); embolization was
performed with permanent materials alone in the remaining four patients. Regular
menstruation returned in the 44 patients who were followed up (for a mean 12.6-month
period), and five patients were known to become pregnant. [65]

B-Lynch and Cho sutures


Recent case series and case reports advocate the use of transmural uterine compression
sutures to rapidly control bleeding. The initial reports described the B-Lynch technique, which
involves opening the lower segment and passing a suture through the posterior uterine wall
and then over the fundus to be tied anteriorly. [66, 67] A similar technique has been described
without opening the uterus. A long, straight needle is passed anterior to posterior through the
lower uterine segment; the suture is passed over the fundus and then tied anteriorly. [68] Both
techniques use bilateral stitches. The most recent variant uses multiple stitches passed
transmurally and tied anteriorly at various points over the uterine body. This technique may be
focused in the area of the placental bed in cases of abnormal placentation. [69] All of these
procedures effectively produce tamponade by compressing together the anterior and
posterior walls.

Follow-up reports suggest a normal return to menses and fertility, but the number of cases is
small. The techniques have the advantage of being very simple to perform and may be a
rapidly effective alternative to hysterectomy. [66]

Bleeding at cesarean delivery


In the past, most cases of intractable PPH followed vaginal delivery and were due to uterine
atony; however, more recent case series and national databases show that more cases are
now associated with cesarean delivery. Cesarean delivery for placenta previa carries a
relative risk of 100 for peripartum hysterectomy, with many patients having a diagnosis of
placenta accreta.[70] High-resolution ultrasound with color Doppler may allow antenatal
diagnosis of placenta accreta.

Whenever possible, delivery of the placenta at cesarean delivery should be performed in an


assisted fashion following the administration of a uterotonic agent, preferably oxytocin. This
practice leads to less blood loss and less infectious morbidity. [71, 72]

Uterine rupture has also become a more common cause of severe PPH necessitating
hysterectomy. The vast majority of these cases occur in patients with a previous cesarean
birth. Counsel all women with placenta previa, and especially those with a previous low
segment uterine scar, in the antenatal period regarding the risk of severe PPH and the
possible need for transfusion and even hysterectomy. Ensure that these patients are cared for
in facilities with the resources to manage them successfully if complications arise. [73]

The management of bleeding at cesarean delivery or following uterine rupture is not greatly
different from that following vaginal delivery. Aggressive resuscitation is performed with
attention to restoration of circulating volume and oxygen-carrying capacity and correction of
hemostatic defects. Direct bimanual compression may be used in the case of atony. Retained
tissue may be removed under direct visualization. Abnormally adherent tissue is a concern;
leave it in situ if it cannot be easily removed.

Direct intramyometrial injection of uterotonics may be undertaken. Vasopressin (0.2 U in 1 mL


of NS) may also be injected into the myometrium, with great care taken to avoid intravascular
injection. Individual vessels in the placental bed may be ligated. Simple or box stitches may
be placed where continuous oozing is present.[69] In cases of placenta previa, the lower uterine
segment may be temporarily packed; leaving a pack in the uterus is also an option. The end
of the pack is fed through the cervix and into the vagina and is removed 24-36 hours later.
Uterine rupture or extension of a uterine incision requires excellent visualization and careful
repair with attention to adjacent structures.

The stepwise surgical approach described above may be used if these measures are
unsuccessful and preservation of fertility is desired. Strongly consider immediate
hysterectomy if further reproduction is not an issue or if bleeding or damage to the uterus
appears severe. Embolization may be considered in this setting. Its successful use has been
described both intraoperatively to preserve the uterus and after hysterectomy for continued
bleeding. Embolization may also be used for continued postoperative vaginal bleeding. [74]

Persistent bleeding following hysterectomy may also be managed by packing with gauze
brought out through the vagina or by a pelvic pressure pack composed of gauze in a sterile
plastic bag brought out through the vagina and placed under tension. This pack is also known
as a parachute, mushroom, or umbrella pack. Place a Foley catheter to monitor urine output
and prevent urinary retention. The placement of a suction drain may be useful to monitor
losses in cases of ongoing oozing. Always consider coagulopathy in patients with continued
slow blood loss.

Postoperative Details
Continue resuscitation, and repeat laboratory tests. Monitor vital signs, urine output, and any
ongoing losses. Care in an intensive care setting is advantageous, as is close follow-up by
the obstetric service. The patient must be monitored for complications (see Complications).
Follow-up
Full documentation of the case is imperative, and a careful explanation of events and
interventions must be given to the patient and family. Caregivers must be available and
approachable for questions. Implications and recommendations for future pregnancies may
be discussed during the postoperative stay and reinforced at the postdischarge visit.

Summary
PPH is a common complication of childbirth and a leading cause of maternal morbidity and
mortality. Clinicians should identify risk factors before and during labor so that care may be
optimized for high-risk women. However, significant life-threatening bleeding can occur in the
absence of risk factors and without warning. All caregivers and facilities involved in maternity
care must have a clear plan for the prevention and management of PPH. This includes sound
resuscitation skills and familiarity with all medical and surgical therapies available.

Complications
Most patients with PPH are quickly identified and successfully treated before major
complications develop. The most common problem is anemia and loss of iron stores, which
results in fatigue in the postpartum period. Clinicians and patients are more tolerant of low
hemoglobin levels, mild postural lightheadedness, and fatigue because of current concerns
over blood transfusion. The risks of transfusion with blood products are well known and have
been previously described.

Not surprisingly, many of the complications of severe PPH are related to massive blood loss
and hypovolemic shock. Damage to all major organs is possible; respiratory (adult respiratory
distress syndrome) and renal (acute tubular necrosis) damage are the most common but are
rare. These conditions are best managed by specialists. Renal failure is usually self-limited,
and renal function recovers fully. Temporary dialysis is seldom required. Pulmonary edema is
uncommon in this previously healthy group; however, it may develop acutely or during the
recovery phase because of fluid overload or myocardial dysfunction. Response to standard
therapy is usually prompt.

Pregnant women are at increased risk of venous thrombosis and embolic events. Many of the
risk factors for PPH are also risk factors for venous thrombosis and embolic events, including
operative vaginal delivery, cesarean delivery, and pelvic surgery. Venous stasis due to shock
and immobility also contribute, and caregivers should maintain a high index of clinical
awareness.

Hypopituitarism following severe PPH (Sheehan syndrome) is due to critical ischemia of the
hypertrophied pituitary. This condition should be considered if a failure to lactate occurs.
Isolated deficiencies of pituitary tropins and hyperprolactinemia have also been reported.

Evidence suggests that prophylaxis against gastrointestinal ulceration is useful in critically ill
patients, especially those requiring ventilation. The recommended agents are sucralfate and
histamine 2 blockers. Both are effective at reducing the risk of ulcers. Sucralfate may be
associated with a lower incidence of pneumonia.[75]

Several of the complications related to surgical interventions have been described.


Complications include sterility, uterine perforation, uterine synechiae (Asherman syndrome),
urinary tract injury and genitourinary fistula, bowel injury and genitointestinal fistula, vascular
injury, pelvic hematoma, and sepsis. Consider ultrasound of the kidneys following complicated
emergency pelvic surgery in order to exclude ureteric obstruction. Patients undergoing uterine
exploration, instrumentation, or laparotomy in this context probably benefit from antibiotic
coverage at the time of the intervention. Good evidence suggests that all patients having
cesarean births should receive prophylactic antibiotics. [76] The duration of antibiotic coverage
following surgery in these circumstance is unknown.

http://emedicine.medscape.com/article/275038-treatment#showall
http://apps.who.int/iris/bitstream/10665/75411/1/9789241548502_eng.pdf

Treatment
Initial therapy includes oxygen delivery, bimanual massage, removal of any blood clots from
the uterus, emptying of the bladder, and the routine administration of dilute oxytocin infusion
(10-40 U in 1000 mL of lactated Ringer solution [LRS] or isotonic sodium chloride solution). If
retained products of conception are noted, perform manual removal or uterine curettage.

If oxytocin is ineffective, carboprost in an intramuscularly administered dose of 0.25 mg can


be administered every 15 minutes, not to exceed 3 doses. Studies indicate a 75-88% success
rate when carboprost is used alone and a 95% success rate when it is used in combination
with other oxytocic agents.

Methylergonovine can also be intramuscularly administered in a dose of 0.2 mg. Because this
agent causes intense vasoconstriction and may cause transient hypertension, it is
contraindicated in patients with hypertensive disease. Check blood pressure prior to
administration.
Misoprostol has been used clinically for the treatment of postpartum hemorrhage. [5, 6] However,
further research is needed to determine the effectiveness, optimal dosage, and route of
administration.[5, 6] A 2010 Monte Carlo simulation indicated that both sublingual and
prophylactic oral misoprostol lowered mortality but raised costs (estimated incremental costs
per disability-adjusted life year were $6 and $170, respectively). [7] Also, a 2010 prospective,
randomized, double-blind trial of rectal misoprostol vs intravenous oxytocin to prevent blood
loss after cesarean delivery revealed that blood loss was significantly lower in the misoprostol
group.[8]

When postpartum hemorrhage is not responsive to pharmacological therapy and no vaginal


or cervical lacerations have been identified, consider the following more invasive treatment
methods:

Uterine packing is now considered safe and effective therapy for the
treatment of postpartum hemorrhage.[9] Use prophylactic antibiotics and concomitant
oxytocin with this technique. The timing of removal of the packing is controversial,
but most physicians favor 24-36 hours. This treatment is successful in half of
patients. If unsuccessful, it still provides time in which the patient can be stabilized
before other surgical techniques are employed.
A Foley catheter with a large bulb (24F) can be used as an alternative to
uterine packing.[10] This technique can be highly effective, is inexpensive, requires no
special training, and may prevent the need for surgery.[10]
Uterine artery embolization, which is performed under local anesthesia, is a
minimally invasive technique.[11] The success rate is greater than 90%.[12]This
procedure is believed to preserve fertility.[12, 11] Complications are rare (6-7%) and
include fever, infection, and nontarget embolization. In patients at high risk for
postpartum hemorrhage, such as those with placenta previa, placenta accreta,
coagulopathy, or cervical pregnancy, the catheter can be placed prophylactically.[12]
The B-Lynch suture technique:[13] A suture is passed through the anterior
uterine wall in the lower uterine segment approximately 3 cm medial to the lateral
edge of the uterus. The suture is wrapped over the fundus 34 cm medial to the
cornual and inserted into the posterior uterine wall again in the lower uterine
segment approximately 3 cm medial to the lateral edge of the uterus and brought
out 3 cm medial to the other edge of the uterus. The suture is wrapped over the
fundus and directed into and out of the anterior uterine wall parallel to the previous
anterior sutures. The uterus is compressed in an accordionlike fashion and the
suture is tied across the lower uterine segment. The B-Lynch suture technique and
other compression suture techniques are operative approaches to postpartum
hemorrhage that have proven to preserve fertility.[14, 15] As practitioners become
proficient in this technique, it may be considered before uterine artery or
hypogastric artery ligation andhysterectomy.
When conservative therapy fails, the next step is surgery with either bilateral uterine artery
ligation or hypogastric artery ligation. Uterine artery ligation is thought to be successful in 80-
95% of patients. If this therapy fails, hypogastric artery ligation is an option. However, this
approach is technically difficult and is only successful in 42-50% of patients. [16] Instead,
stepwise devascularization of the uterus is now thought to be the next best approach, with
possible ligation of the utero-ovarian and infundibulopelvic vessels. [17]

When all other therapies fail, emergency hysterectomy is often a necessary and lifesaving
procedure.

http://emedicine.medscape.com/article/260187-overview#aw2aab6b4

5. Mengapa pada px PPv didapatkan berwarna merah bercampur cairan kuning


keruh dan berbau?

Gynecologic diseases can be judged by the color of leucorrhea:


1. White curd-like leucorrheacolpitis mycotica: white curd-like leucorrhea and extremely itching.

2. Yellow leucorrhea-- trichomonas vaginitis: secretion turns to be yellow or yellow-green, smelly with

bulb and vaginal Burning sensation with vulva itching. All these symptoms are the indication of

trichomonad.

3. Brown leucorrhea--chronic cervicitis or cervical carcinoma: leucorrhea turns to be yellow-sticky or

pus-snot with blood streak, which may be chronic cervicitis. Whether cervicitis will pathology or not is

determined by many factors. Research shows that the incident of female suffering from chronic cervicitis

is 7 times higher than that of normal ones.

6. Apa saja faktor yg memungkan terjadinya kasus spt di skenario?

Postpartum hemorrhage is defined as excessive blood loss during or after the third stage of
labor. The average blood loss is 500 mL at vaginal delivery and 1000 mL at cesarean delivery.
Since diagnosis is based on subjective observation, it is difficult to define clinically.

Objectively, postpartum hemorrhage is defined as a 10% change in hematocrit level between


admission and the postpartum period or the need for transfusion after delivery secondary to
blood loss.[4]

Early postpartum hemorrhage is described as that occurring within the first 24 hours after
delivery. Late postpartum hemorrhage most frequently occurs 1-2 weeks after delivery but
may occur up to 6 weeks postpartum.

Etiology
Early postpartum hemorrhage may result from uterine atony, retained products of conception,
uterine rupture, uterine inversion, placenta accreta, lower genital tract lacerations,
coagulopathy, and hematoma. Causes of late postpartum hemorrhage include retained
products of conception, infection, subinvolution of placental site, and coagulopathy.

Uterine atony and lower genital tract lacerations are the most common causes of postpartum
hemorrhage. Factors predisposing to uterine atony include overdistension of the uterus
secondary to multiple gestations, polyhydramnios, macrosomia, rapid or prolonged labor,
grand multiparity, oxytocin administration, intra-amniotic infection, and use of uterine-relaxing
agents such as terbutaline, magnesium sulfate, halogenated anesthetics, or nitroglycerin. In
uterine atony, lack of closure of the spiral arteries and venous sinuses coupled with the
increased blood flow to the pregnant uterus causes excessive bleeding.

Active management of the third stage of labor with administration of uterotonics before the
placenta is delivered (oxytocin still being the agent of choice), early clamping and cutting of
the umbilical cord, and traction on the umbilical cord have proven to reduce blood loss and
decrease the rate of postpartum hemorrhage.

Lower genital tract lacerations, including cervical and vaginal lacerations (eg, sulcal tears),
are the result of obstetrical trauma and are more common with operative vaginal deliveries,
such as with forceps or vacuum extraction. Other predisposing factors include macrosomia,
precipitous delivery, and episiotomy.

http://emedicine.medscape.com/article/260187-overview#aw2aab6b4

Organisme yang menyerang bekas implantasi plasenta atau laserasi akibat persalinan
adalah penghuni normal serviks dan jalan lahir, mungkin juga dari luar. Biasanya
lebih dari satu spesies. Kuman anaerob adalah kokus gram positif (peptostreptokok,
peptokok, bakteriodes dan clostridium). Kuman aerob adalah berbagai macam gram
positif dan E. coli. Mikoplasma dalam laporan terakhir mungkin memegang peran
penting sebagai etiologi infeksi nifas.

Bermacam-macam jalan kuman masuk ke dalam alat kandungan, seperti eksogen


(kuman datang dari luar), autogen (kuman masuk dari tempat lain dalam tubuh), dan
endogen (dari jalan lahir sendiri). Penyebab yang terbanyak dan lebih dari 50%
adalah streptococcus anaerob yang sebenarnya tidak patogen sebagai penghuni
normal jalan lahir.
Kuman-kuman yang sering menyebabkan infeksi antara lain adalah:
(1) Streptococcus haemoliticus aerobik
Masuknya secara eksogen dan menyebabkan infeksi berat yang ditularkan dari penderita
lain, alat-alat yang tidak suci hama, tangan penolong, dan sebagainya.
(2) Staphylococcus aureus
Masuk secara eksogen, infeksinya sedang, banyak ditemukan sebagai penyebab
infeksi di rumah sakit.
(3) Escherichia coli
Sering berasal dari kandung kemih dan rektum, menyebabkan infeksi terbatas.
(4) Clostridium welchii
Kuman anaerobik yang sangat berbahaya, sering ditemukan pada abortus kriminalis
dan partus yang ditolong dukun dari luar rumah sakit.
Secara umum frekuensi infeksi puerperalis adalah sekitar 1-3%. Secara proporsional
angka infeksi menurut jenis infeksi adalah:
- Infeksi jalan lahir 25 sampai 55% dari kasus infeksi
- Infeksi saluran kencing 30-60% dari kasus infeksi
- Infeksi pada mamma 5-10% dari kasus infeksi
- Infeksi campuran 2-5% dari kasus infeksi.

(adneksitis)
Adneksitis adalah Infeksi / radang pada Adnexsa rahim. Adneksa adalah jaringan yang
berada di sekitar rahim. Ini termasuk tuba fallopi dan ovarium alias indung
telur,tempat dimana sel telur diproduksi. Istilah lain yang sering digunakan untuk
menyebut Adnexitis adalah PID (Pelvic Inflammatory Disease) atau Salpingitis. Dari
pengertian tersebut, dapat ditarik kesimpulan bahwa Adnexitis hanya menyerang
kaum wanita, karena merekalah yang memiliki rahim, sedangkan pria tidak.Penyakit
ini dapat membawa dampak yang serius jika tidak segera ditangani, seperti
kemandulan, kehamilan diluar rahim, keluarnya nanah dari vagina, dan nyeri
panggulkronis.

Karena ada infeksi o/ streptococcus anaerob


Tipe infeksi endogen (penyebaran dari jalan lahir)
Bakteri yg menyebabkan infeksi post partum:
a. Streptococcus b hemolitikus (eksogen, ditularkan dari penderita lain)
b. Staphylococcus aureus (eksogen, ditemukan sbg penyebab infeksi
nosokomial)
c. E coli (berasal dari VU dan rectum)
d. Clostridium welchii (kuman anaerob berbahaya, ditemukan pada
abortus criminalis dan partus yang ditolong dukun)

7. Mengapa dr mengusulkan obat parasetamol dan pasien banyak minum?

Parasetamol (asetaminofen) merupakan obat analgetik non narkotik dengan


cara kerja menghambat sintesis prostaglandin terutama di Sistem Syaraf
Pusat (SSP) . Parasetamol digunakan secara luas di berbagai negara baik
dalam bentuk sediaan tunggal sebagai analgetik-antipiretik maupun kombinasi
dengan obat lain dalam sediaan obat flu, melalui resep dokter atau yang dijual
bebas. (Lusiana Darsono 2002)
Parasetamol adalah paraaminofenol yang merupakan metabolit fenasetin dan
telah digunakan sejak tahun 1893 (Wilmana, 1995). Parasetamol
(asetaminofen) mempunyai daya kerja analgetik, antipiretik, tidak mempunyai
daya kerja anti radang dan tidak menyebabkan iritasi serta peradangan
lambung (Sartono,1993).
Parasetamol, mempunyai daya kerja analgetik dan antipiretik sama dengan
asetosal, meskipun secara kimia tidak berkaitan. Tidak seperti Asetosal,
Parasetamol tidak mempunyai daya kerja antiradang, dan tidak menimbulkan
iritasi dan pendarahan lambung. Sebagai obat antipiretika, dapat digunakan
baik Asetosal, Salsilamid maupun Parasetamol.
Farmakokinetik
Parasetamol cepat diabsorbsi dari saluran pencernaan, dengan kadar serum
puncak dicapai dalam 30-60 menit. Waktu paruh kira-kira 2 jam. Metabolisme
di hati, sekitar 3 % diekskresi dalam bentuk tidak berubah melalui urin dan 80-
90 % dikonjugasi dengan asam glukoronik atau asam sulfurik kemudian
diekskresi melalui urin dalam satu hari pertama; sebagian dihidroksilasi
menjadi N asetil benzokuinon yang sangat reaktif dan berpotensi menjadi
metabolit berbahaya. Pada dosis normal bereaksi dengan gugus sulfhidril dari
glutation menjadi substansi nontoksik. Pada dosis besar akan berikatan dengan
sulfhidril dari protein hati.(Lusiana Darsono 2002)
Farmakodinamik
Efek analgesik Parasetamol dan Fenasetin serupa dengan Salisilat yaitu
menghilangkan atau mengurangi nyeri ringan sampai sedang. Keduanya
menurunkan suhu tubuh dengan mekanisme yang diduga juga berdasarkan
efek sentral seperti salisilat.
Efek anti-inflamasinya sangat lemah, oleh karena itu Parasetamol dan
Fenasetin tidak digunakan sebagai antireumatik. Parasetamol merupakan
penghambat biosintesis prostaglandin (PG) yang lemah. Efek iritasi, erosi dan
perdarahan lambung tidak terlihat pada kedua obat ini, demikian juga
gangguan pernapasan dan keseimbangan asam basa.(Mahar Mardjono 1971)
Semua obat analgetik non opioid bekerja melalui penghambatan
siklooksigenase. Parasetamol menghambat siklooksigenase sehingga konversi
asam arakhidonat menjadi prostaglandin terganggu. Setiap obat menghambat
siklooksigenase secara berbeda. Parasetamol menghambat siklooksigenase
pusat lebih kuat dari pada aspirin, inilah yang menyebabkan Parasetamol
menjadi obat antipiretik yang kuat melalui efek pada pusat pengaturan panas.
Parasetamol hanya mempunyai efek ringan pada siklooksigenase perifer.
Inilah yang menyebabkan parasetamol hanya menghilangkan atau mengurangi
rasa nyeri ringan sampai sedang.
Parasetamol tidak mempengaruhi nyeri yang ditimbulkan efek langsung
prostaglandin, ini menunjukkan bahwa parasetamol menghambat sintesa
prostaglandin dan bukan blokade langsung prostaglandin. Obat ini menekan
efek zat pirogen endogen dengan menghambat sintesa prostaglandin, tetapi
demam yang ditimbulkan akibat pemberian prostaglandin tidak dipengaruhi,
demikian pula peningkatan suhu oleh sebab lain, seperti latihan fisik. (Aris
2009)
Indikasi
Parasetamol merupakan pilihan lini pertama bagi penanganan demam dan
nyeri sebagai antipiretik dan analgetik. Parasetamol digunakan bagi nyeri yang
ringan sampai sedang.(Cranswick 2000)
Kontra Indikasi
Penderita gangguan fungsi hati yang berat dan penderita hipersensitif terhadap
obat ini. (Yulida 2009)

8. Apakah pada px abdomen interpretasi normal?

Proses INVOLUSI UTERI adalah sbb :


AUTOLYSIS
Autolysis merupakan proses penghancuran diri sendiri yang terjadi didalam
otot uterine. Enzym proteolitik akan memendekkan jaringan otot dan jaringan
ikat yang telah sempat mengendur hingga 10 kali panjangnya dari semula dan
5 kali lebarnya dari semula selama kehamilan, sehingga uterus akan berangsur
angsur mengecil.
EFEK OKSITOSIN
Kontraksi dan retraksi otot uterine akan mengkompres pembuluh darah dan
oleh karena itu akan mengurangi suplai darah ke uterus. Proses ini berguna
bagi pengurangan situs atau tempat implantasi plasenta serta pengurangan
perdarahan
Proses Involusi pd bekas Implantasi plasenta placental bed mengecil krn
kontraksi & menonjol ke kavum uteri dgn diameter 7,5 cm. sesudah 2 mg mjd
3,5 cm, pd mg ke-6 mjd 2,4 cm dan akhirnya pulih
Afterpains krn kontraksi, biasanya berlangsung 2-4 hari pasca salin. Rasa
nyeri stlh melahirkan akan lebih nyata stlh ibu melahirkan bayi besar atau
kembar dan pada multipara
Perlu diberikan pengertian pd ibu dan dpt diberikan obat antisakit
dan antimules
Involusi Tinggi Fundus Berat Utrs
Bayi lahir Sepusat 1000
Plasenta lahir 2 jari bawah pusat 750 gr
7 hari(1 mg) Pertengahan pst-symp 500 gr
14 hari(2 mg) Tak teraba di atas symp 350 gr
42 hari(6 mg) Bertambah kecil 50 gr
56 hari(8 mg) Normal 30 gr

*Stlh bbrp hari pasca persalinan, perubahan involusi berlangsung cepat.


*Fundus turun kira-kira 1-2 cm setiap 24 jam
(Bobak,2005 :493)

Tinggi fundus segera setelah placenta lahir, tinggi fundus setingi pusat,
kemudian berangsur mengecil. Kalau tinggi fundusnya masih sama seperti saat
melahirkan (di atas pusat) curigai atonia uteri. Kalau perutnya teraba keras dan pasien sangat
kesakitan berarti dicurigai ruptur uteri.

Lembek nya kenapa?

Atonia Uteri

Keadaan lemahnya tonus atau kontraksi rahim yang menyebabkan uterus tidak
mampu menutup perdarahan terbuka dari tempat implantasi plasenta setelah bayi dan
plasenta lahir.

Diagnosa:

Ditegakkan bila setelah bayi dan plasenta lahir ternyata perdarahan masih aktifdan
banyak, mengumpal dan pada palpasi didapatkan fundus uteri masih setinggi pusat
atau lebih dengan kontraksi yang lembek. Perlu diperhatikan bahwa pada saat atonia
uteri didiagnosa, maka pada saat itu juga masih ada darah sebanyak 500-1000 cc yang
keluar dari pembuluh darah, tetapi masih terperangkap dalam uterus.

Ilmu kebidanan Sarwono Prawirohardjo

9. Interpretasi pada VT px ginekologis?


10. Px lab apa yang diusulkan?
11. Kelainan-kelainan pada masa nifas?

Ada 4 masalah yang mungkin timbul pada masa nifas:

a. Perdarahan post partum


- Yaitu perdarahan per vaginam > 500ml yang terjadi dalam 24 jam pertama
setelah melahirkan post partum primer
- Atau pada masa nifas setelah 24 jam disebut perdarahan post partum
sekunder

Etiologi

Normalnya stlh melahirkan bayi. Tjd kehilangandarah sebanyak 200-


400ml sebelum retraksi miometrium dan dilengkapi uterus yang kuat
menyebabkan pemendekan dan penyempitan pembuluh darah uterus dan
retraksi jaringan plasenta perubahan ini mencegah perdarahan lebih
lanjut
Jika uterus tdk berkontraksi secara efekti ( atonia uteri) / jika sisa plasenta
mencegah retraksi tempat plasenta secara baik , perdarahan dpt terjadi 2
penyebab ini menjadi dan bertanggung jawab atas 80% kasus PPP
20% kasus karena laserasi traktus genitalis, biasanya vagina / serviks, tapi
jarang juga karena rupture uterus
Perdarahan kala 3 ( plasenta masih di dalam uterus )
Perdarahn postpartum sejati ( placenta sudah keluar )
Perdarahan post partum sekundr mencakup semua kejadian PPH yang
terjadi antara 24 jam setelah kelahiran bayi dan 6 minggu masa post
partum. ( penyebabnya adalah epitelisasi yang buruk pada t4 plasenta
sekitar 80%, fragmen plasenta dan / bekuan darah yang tertahan dalam
uterus

b. Infeksi masa nifas


Infeksi nifas adalah infeksi luka pada jalan lahir setelah melahirkan, yang
kadang kala meluas, menyebabkan flebitis atau peritonitis (Reeder, 2011).

Kenaikan suhu tubuh sampai 38 derajat / lebih yang berlangsung selama


24 jam atau kambuh kembali sejak akhir hari 1 akhir hari 10setelah
melahirkan atau setelah abortus

Macam2 jalan kuman masuk ke dalam alat kandungan

- Eksogen (kuman datang dari luar)


- Autogen (kuman masuk dari tempat lain dalam tubuh)

- Endogen (dari jalan lahir sendiri)

- Paling banyak kasus ineksi berasal dari infeksi ascenderens


ETIOLOGI :
Yang paling terbanyak dan lebih dari 50% : streptococcus anaerob yang
sebenarnya tidak pathogen sebagai penghuni normal jalan lahir
Kuman yang sering menyebabkan infeksi antara lain :
1. Streptococcus haemoliticus aerobic
Masuknya secara eksogen dan menyebabkan infeksi berat yang ditularkan dari
penderita lain, alat2 yang tidak suci hama, tangan penolong, dan sebagainya.
2. Staphylococcus aureus
Masuk secara eksogen, infeksinya sedang, banyak ditemukan sebagai penyebab
infeksi di rumah sakit.
3. Escherichia coli
Sering berasal dari kandung kemih dan rectum, menyebabkan infeksi terbatas.
4. Clostridium welchii
Kuman anaerobic yang sangat berbahaya, sering ditemukan pada abortus kriminalis
dan partus yang ditolong dukun dari luar rumah sakit.

c. Tromboemboli
Trombosis pada sebuah vena yg dpt terjadi saat kehamilan/ lebih sering terjadi
pada masa nifas antara hari 5 dan 15
Terjadi pada wanita gemuk, usia > 35 tahun, dan punya riwayan seksio sesaria
sebelumnya
Biasanya dimulai dari vena profunda tungkai bawah
Diagnosis Trombosis vena profunda pireksi derajat rendah , denyut nasi
meningkat dan gelisah
d. Masalah psikatri post partum
Murung 3 hari 50-70% ibu mengalami emosi labil, mudah tersinggung,
dimulai hari ketiga dan kelima setelah kelahiran bayi emosi labil biasanya
kurang dari 1 minggu
Depresi postnatal 8-12% wanita mengalami depresi klinis pada 3 bulan
pertama stlah melahirkan . Terjadi karena mempunyai masalh social dengasn
peristiw akehidupan penuh dengan stres

Prof. Dr. Rustam Mochtar, MPH. Editor : Dr. Delfi Lutan, DSOG. Sinopsis
Obstetri : Obstetri Fisiologi Obstetri Patologi. Jillid 1. Edisi 2. EGC , Derek
Llewellyn-Jones, Obstetri dan Ginekologi Ed.6

Infeksi nifaskenaikan suhu badan sampai 38oC atau lebih selama 2 hari dalam 10
hari pertama postpartum, kecuali pada hari pertama.
Prof. Dr. Rustam Mochtar, MPH. Editor : Dr. Delfi Lutan, DSOG. Sinopsis
Obstetri : Obstetri Fisiologi Obstetri Patologi. Jillid 1. Edisi 2. EGC

Macam-macam infeksi nifas:


a) Vulvitis
Pada infeksi bekas sayatan episiotomi atau luka perineum sekitarnya membengkak,
tepi luka menjadi merah dan bengkak, jahitan mudah terlepas, dan luka yang terbuka
menjadi ulkus dan mengeluarkan pus.
b) Vaginitis
Infeksi vagina dapat terjadi secara langsung pada luka vagina atau melalui perineum.
Permukaan mukosa membengkak dan kemerahan, terjadi ulkus, dan getah
mengandung nanah yang keluar dari ulkus. Penyebaran dapat terjadi, tetapi pada
umumnya infeksi tinggal terbatas.
c) Servisitis
Infeksi servik juga sering terjadi, akan tetapi biasanya tidak menimbulkan banyak
gejala. Luka servik yang dalam, meluas, dan langsung ke dasar ligamentum latum
dapat menyebabkan infeksi yang menjalar ke parametrium.
d) Endometritis
Jenis infeksi yang paling sering adalah endometritis. Kuman-kuman memasuki
endometrium, biasanya pada luka bekas insersio plasenta, dan dalam waktu singkat
mengikutsertakan seluruh endometrium. Pada infeksi dengan kuman yang tidak
seberapa patogen, radang terbatas pada endometrium. Jaringan desidua bersama-sama
dengan bekuan darah menjadi nekrotis dan mengeluarkan getah berbau dan terdiri
atas keping-keping nekrotis serta cairan. Pada batas antara daerah yang meradang dan
daerah sehat terdapat lapisan terdiri atas leukosit-leukosit. Pada infeksi yang lebih
berat batas endometrium dapat dilampaui dan terjadilah penjalaran.
e) Septikemia dan piemia
Ini merupakan infeksi umum yang disebabkan oleh kuman-kuman yang sangat
patogen biasanya Streptococcus haemolilyticus golongan A. Infeksi ini sangat
berbahaya dan merupakan 50% dari semua kematian karena infeksi nifas. Adanya
septikemia dapat dibuktikan dengan jalan pembiakan kuman-kuman dari darah. Pada
piemia terdapat dahulu tromboflebitis pada vena-vena di uterus serta sinus-sinus pada
bekas implantasi plasenta.
Tromboflebitis ini menjalar ke vena uterina, vena hipogastrika dan/atau vena ovarii.
Dari tempat-tempat trombus itu embolus kecil yang mengandung kuman-kuman
dilepaskan. Tiap kali dilepaskan, embolus masuk ke dalam peredaran darah umum
dan dibawa oleh aliran darah ke tempat-tempat lain, diantaranya paru, ginjal, otak,
jantung, dan mengakibatkan terjadinya abses-abses di tempat-tempat tersebut.
Keadaan ini dinamakan piemia. streptococci dan H.parainfluenzae. Cedera payudara
mungkin Karena memar karena manipulasi yang kasar, pembesaran payudara, stasis
air susu ibu dalam duktus, atau pecahnya puting susu.
Bakteri berasal dari berbagai sumber diantaranya: tangan ibu, tangan orang yang
merawat ibu atau bayi, bayi, duktus laktiferus, darah sirkulasi. Sedangkan tanda dan
gejala mastitis diantaranya meliputi: peningkatan suhu yang cepat dari 39,5 0C sampai
400C, peningkatan kecepatan nadi, menggigil, malaise umum, sakit kepala, nyeri
hebat, bengkak, inflamasi, area payudara keras.
Prawirohardjo, Sarwono. 2008. ILMU KEBIDANAN. Jakarta: PT Bina Pustaka

12. Apa saja Pencegahan yg dapat dilakukan untuk kasus?


13. DD?

Prescription medications should be reserved for women with frequently recurring


vaginitis who have failed to improve with over-the-counter medication or for patients
with compliance issues for whom shorter, higher-dose treatment schedules may
improve outcome. Terconazole 0.4% cream or 80-mg suppositories nightly for 7 days,
or 0.8% cream nightly for 3 days may be used. In some studies terconazole has a
higher cure rate than over-the-counter medications, perhaps because of its
effectiveness against C. glabrata.10, 43However, other studies have not borne this out.27

Newer 1-day oral regimens for yeast vaginitis are also available including fluconazole
150 mg or itraconazole 200 mg. They are as effective as intravaginal treatment of
vulvovaginal candidiasis, but gastrointestinal toxicity resulting in nausea, vomiting,
and diarrhea is common, occurring in up to 15% of patients.45, 46 One-dose topical
treatments are also available (tioconazole 6.5% vaginal ointment, clotrimazole 500-
mg vaginal suppository), but appear to have less efficacy than other preparations.47

For patients with frequently recurrent candidal vaginitis, it is important to consider


factors that predispose to infection. The discontinuation of oral contraceptives
decreases the frequency of yeast vaginitis for many women.11 Eliminating nylon and
tight-fitting garments can also be helpful.48 Many diabetic patients with poor glycemic
control have recurring yeast vaginitis; however, the routine use of oral glucose
tolerance tests or glycosylated hemoglobin levels is not advocated unless there is
further clinical evidence of diabetes.49 Patients with recurrent yeast vaginitis should,
however, be tested for HIV. In one small study, the daily intake of 8 oz of yogurt
containing live Lactobacillus acidophilus has been shown to decrease candida
colonization and vaginitis.50, 51 Further study is required to determine if this is actually
helpful in the management of recurrent yeast vaginitis. Hyposensitization with
candida antigen has been reported to be effective for women who have an allergic
vaginitis due to candida.52

When conventional treatment measures are ineffective, longer candicide treatment


regimens are necessary, usually for 10 days to 2 weeks. For women with recurrent
infection associated with menstruation, an intravaginal candicide such as clotrimazole
vaginal suppositories, 100 mg nightly for several days prior to menstruation, may be
effective.11 Alternatively, ketoconazole, 200 mg orally twice a day for 5 to 7 days each
month beginning prior to menstruation has been shown to be effective but is
associated with a small risk of hepatotoxicity. A regimen of ketoconazole, 100 mg
daily for 6 months, has only a 5% recurrence rate for yeast infection.53 This low dose
is felt to inhibit germination of yeast.25 In addition to hepatotoxicity, ketoconazole has
significant drug interactions with other frequently used medications. Fluconazole, 150
mg once a week or monthly, has less hepatic toxicity and is a reasonable alternative.
The risks of suppressive therapy should be weighed against the potential benefits,
including the potential for the development of resistant strains of yeast. Monthly use
of an over-the-counter candicide prior to menstruation is preferable to long-term
suppressive therapy for most patients.

BACTERIAL VAGINOSIS

Pathology

Bacterial vaginosis has a complex and poorly understood pathophysiology,54 even


though it is a frequent cause of vaginitis in women.33 The prevalence of this disorder is
underestimated, as 50% of women with bacterial vaginosis are asymptomatic.55 The
nomenclature for this infection has changed in recent years, from nonspecific vaginitis
to bacterial vaginosisthe term vaginosis reflecting the lack of an inflammatory
component. Symptoms of bacterial vaginosis result from overgrowth of certain
bacterial species, concurrent with a decrease in normal flora and lactobacilli. In
addition, strains of lactobacilli that produce hydrogen peroxide are often replaced by
nonproducing strains.55 Ninety-six percent of normal women have hydrogen peroxide
producing Lactobacillus species compared with only 6% of women with bacterial
vaginosis.56 The production of hydrogen peroxide by lactobacilli combines with
chlorine, a halide present in cervical mucus, to produce a nonspecific antimicrobial
host defense. The combination of these substances is toxic to some of the organisms
associated with bacterial vaginosis, including Gardnerella vaginalis and prevotella.
The level of hydrogen peroxide available to combine with chlorine in the vagina may
influence susceptibility to bacterial vaginosis.56

A number of organisms are associated with this infection including G.


vaginalis, Myocoplasma hominis,Mobiluncus species, Bacteroides species other
than B. fragilis, and other anaerobic gram-positive cocci such as prevotella
and Peptostreptococcus.57 The exact mechanism by which these organisms cause
infection is not established.2Gardnerella vaginalis is present in many women without
symptoms of vaginal infection. Carrier rates of this organism in some series are as
high as 10% to 40% of normal women.22 The presence of this organism does not prove
infection 5759; however, in women with bacterial vaginosis, the concentration of G.
vaginalis is two to three logs higher than in normal women.

High rates of bacterial vaginosis have been reported from sexually transmitted disease
clinics, but the role of sexual transmission of bacterial vaginosis is unclear. Many of
the studies are small, and prevalence ranges are broad.4, 6062 Male sexual contacts of
women with bacterial vaginosis have been shown to carry similar
organisms.11 Evidence against sexual transmission, however, includes similar rates
(approximately 15%) of bacterial vaginosis in a population of prostitutes and a
population of college students in Seattle.63,64 Moreover, treatment of sexual partners
does not reduce the risk of recurrence.54 Bacterial vaginosis has also been found in
virginal adolescents.65 Bacterial vaginosis is most likely associated with sexual
transmission through the disruption of normal vaginal flora but is not a sexually
transmitted disease.

The prevalence of bacterial vaginosis has varied with the method of contraception.
The use of an intrauterine device has been associated with a higher rate of bacterial
vaginosis compared with women using other means of contraception (20% vs
6%).60, 61 Women who do not use any form of contraception have also been found to
have a higher rate of bacterial vaginosis.59, 66 The reason for these variations with
contraceptive use is not clear. There is twice the incidence of bacterial vaginosis in
African-American women than white women.67 The higher vaginal pH found in
African-American women 68 may increase their susceptibility to bacterial vaginosis.

Bacterial vaginosis has been associated with a number of complications including an


increased rate of abnormal Pap smears, usually atypical squamous cells of
undetermined significance (ASCUS).69, 70Women with ASCUS should be evaluated for
bacterial vaginosis as they are frequently concurrent problems.69, 70 Bacterial vaginosis
has also been associated with pelvic inflammatory disease (PID).71Eschenbach found a
higher rate of adnexal tenderness as well as a clinical diagnosis of PID in women with
bacterial vaginosis than in those without bacterial vaginosis attending a sexually
transmitted disease clinic.4 The microorganisms of bacterial vaginosis are commonly
isolated from the upper genital tract of patients with acute salpingitis, and bacterial
vaginosis is a frequent concurrent diagnosis in patients with PID.72 The association of
bacterial vaginosis with PID is controversial, and other studies have disputed bacterial
vaginosis as an etiology of PID.73 The finding of an increased incidence of plasma cell
endometritis in women with bacterial vaginosis is supportive of an association
between bacterial vaginosis and upper genital tract infection. The presence of plasma
cells is a marker for endometritis or upper genital tract disease. Plasma cell
endometritis in one study was distinctly associated with bacterial vaginosis, but not
with gonorrhea or chlamydia infection.74

Premature rupture of membranes and preterm delivery of low-birth-weight infants has


also been reported with bacterial vaginosis in several case-control and cohort
studies.7578 Up to a 7.3-fold increased risk (95% confidence interval 1.8, 29.4) for
premature rupture of membranes in women with bacterial vaginosis has been
found.79 Treatment of bacterial vaginosis with oral metronidazole in patients with prior
preterm birth has been shown to reduce prematurity (18% vs 39%) and premature
rupture of membranes (5% vs 33%).80 Furthermore, antibiotic treatment including
tetracycline, erythromycin, and clindamycin has been shown to increase newborn
weight among treated mother-infant pairs.81 This may reflect treatment of ureaplasma
urealyticum rather than bacterial vaginosis. Bacteroides has been isolated from the
vaginal cultures of women who have infants weighing less than 2,500 g and who are
more likely to give birth before 37 weeks' gestation.82, 83 Among women with bacterial
vaginosis, the highest risk of preterm delivery of a low-birth-weight infant was in
women who had vaginal cultures positive for bothBacteroidesMycoplasma
hominis.84 In addition, treatment with metronidazole of asymptomatic pregnant
women at risk of preterm delivery who have bacterial vaginosis in the second
trimester reduces the incidence of preterm delivery.85

Postpartum endometritis following cesarean section occurs at five times the rate in
women with bacterial vaginosis as in women without this diagnosis. Organisms
including G. vaginalis, facultative bacteria, and anaerobes recovered from the
endometrial cavity of 60% of women with postpartum endometritis were associated
with bacterial vaginosis.80, 83, 86 Postpartum wound infection after cesarean section may
also occur more commonly in women with bacterial vaginosis.54 Because of these
associated complications, accurate diagnosis of bacterial vaginosis is increasingly
important in pregnancy and prior to pelvic surgery.

Clinical Presentation

Women with bacterial vaginosis have an extremely variable clinical presentation. The
vaginal discharge is frequently the most noticeable symptom. Patients are particularly
troubled by the fishy amine odor of the discharge, which is thin, homogeneous, and
gray to white in color.11 Many patients have no symptoms.55The clinical examination is
marked by the lack of vulvar and groin involvement and normal-appearing vaginal
rugations.

Diagnosis

The traditional diagnosis of bacterial vaginosis has been clinical, requiring three of
four criteria in the evaluation of a vaginal discharge (Table 1). These criteria include
(1) a pH of 4.5 or higher, (2) an amine or fishy odor with the addition of potassium
hydroxide to the discharge (positive Whiff test), (3) a thin homogeneous discharge,
and (4) the presence of clue cells on microscopy.57 A clue cell is a squamous epithelial
cell whose border is obscured by adherent bacteria (Fig. 2).22 The presence of clue
cells indicates greater numbers of organisms, including Gardnerella, which is thought
to be indicative of infection.23 The normal saline wet mount should utilize a small
amount of discharge with copious amounts of saline to disperse the epithelial cells and
facilitate the finding of clue cells.2 The increase in vaginal pH and fishy odor that
characterizes this discharge results from the proliferation of anaerobic bacteria and the
resultant production of amines and vaginal organic acids.78 The addition of 10%
potassium hydroxide intensifies the fishy odor when it is added to vaginal secretions
by converting the amines to a more volatile state.2 The use of gas-liquid
chromatography of vaginal secretions to diagnose bacterial vaginosis is also based on
this phenomenon.86, 87 The discharge usually contains very few white cells although the
presence of greater numbers of white cells does not exclude the diagnosis. A pH less
than 4.5, however, is not consistent with a clinical diagnosis of bacterial vaginosis.2, 4

Figure 2

Epithelial cell and clue cell. 1000. Normal saline wet preparation showing clue
cell (R) with obvious granulation within the epithelial cell and obliteration of the cell
border.

Interclinician variability in making the diagnosis of bacterial vaginosis and the


association with upper reproductive tract complications makes accurate diagnosis of
this condition more vital.2, 5, 57 This has spurred investigators to evaluate other methods
of diagnosis for bacterial vaginosis. The gram stain appears to be better than gas-
liquid chromatography, the proline aminopeptidase test, or vaginal cultures in
predicting infection.2, 57 The Spiegel criteria base the diagnosis of bacterial vaginosis
on the gram stain. The diagnosis requires the presence of fewer than
five Lactobacillus morphotypes, five or moreGardnerella morphotypes, and five or
more other morphotypes (gram-positive cocci, small gram-negative rods, curved
gram-variable rods, or fusiforms) per high-power field.88 The Vaginal Infection and
Prematurity Study Group developed a more widely utilized method to diagnose
bacterial vaginosis using the gram stain, the Nugent system.5 A scoring system from 0
to 10 grades the severity of bacterial vaginosis using the most reliable bacterial
morphotypes to create a summary score of the alteration in vaginal flora. A score of 0
to 3 represents normal flora, 4 to 6 is indeterminate, and 7 to 10 is diagnostic of
bacterial vaginosis.5 The Nugent system has become the gold standard for
diagnosing bacterial vaginosis.

The use of a vaginal culture to diagnose bacterial vaginosis is not advocated.


However, in clinical practice a vaginal culture is frequently obtained from the patient
with symptoms when a conclusive diagnosis cannot be obtained from the wet mount
preparations during the office visit. A culture result of absent normal flora and
abundant gardnerella can be helpful in pointing to a diagnosis of bacterial vaginosis in
such patients. However, a vaginal smear is far more useful and cost-effective and
should always be obtained before a vaginal culture is considered.

Treatment

Until recently, the standard treatment of symptomatic disease was metronidazole, 500
mg orally twice a day or 250 mg orally three times a day for 7 days, resulting in a cure
rate of 80% to 90% (Table 2).55, 63Alternatively, a single oral 2-g dose of metronidazole
can be given, which results in lower rates of secondary yeast infection and equal
initial cure rates of 80% to 90%.89 The shorter-duration regimen, however, may have a
higher recurrence rate.55 For compliant patients, one of the longer treatment courses is
preferable and has greater efficacy than the single-dose regimen.

The availability of topical preparations has obviated the need for systemic therapy in
many patients. Metronidazole 0.75% vaginal gel twice a day for 5 days has the
advantage of fewer systemic reactions. Other regimens include clindamycin 2%
cream vaginally for 7 nights,90 and clindamycin 300 mg orally twice a day for 7 days.
Their efficacy appears to be high93% at 1 week, and 89% at 1 month.91, 92However,
the oral form may result in a higher incidence of diarrhea and colitis. The topical
preparations are only minimally absorbed, reaching 2% to 5% of the mean peak serum
concentration of systemic therapy, resulting in much lower toxicity. Ampicillin and
doxycycline are not as effective in treating bacterial vaginosis (cure rates 40%60%)
and generally should not be used.63, 93 Topical sulfonamide creams have been used for
many years in the therapy of bacterial vaginosis, but there are no data to support their
use. Two randomized, double-blind trials have not shown a benefit from treating the
sexual partner,94,95 and this is not recommended.

Among women treated with metronidazole, 20% to 30% will have a recurrence in 3
months.52 Although metronidazole is highly active against anaerobic bacteria, it is less
active against G. vaginalis and poor against Mobiluncus species. Despite this, a
repeated 7-day course of metronidazole is often effective treatment for a recurrence. A
small number of women will have multiple recurrences. Treatment for these patients
is not established. However, prolonged oral metronidazole use is associated with a
risk of peripheral neuropathy,55 and this must be considered in patients requiring
repetitive therapy. Because of its very limited systemic absorption, this is unlikely to
be a problem with topical metronidazole.
The treatment of bacterial vaginosis during pregnancy is metronidazole, 250 mg
orally three times a day for 7 days, or 2 g orally in a single dose.96 Lower doses are
recommended to limit the exposure of the fetus to medication. The effect of
metronidazole on the fetus during any stage of pregnancy has been a matter of
concern. A 1995 meta-analysis of seven prospective studies and one retrospective
study of first-trimester exposure to metronidazole found no increase in the risk of
teratogenesis from such exposure.97Clindamycin, 300 mg orally twice a day for 7
days, can also be used in pregnancy.96, 98 However, clindamycin vaginal cream is not
recommended because of the finding of an increase in preterm birth in women treated
with clindamycin cream.96, 99 Topical treatments for bacterial vaginosis in pregnancy
may not be adequately effective to reduce preterm delivery, and systemic therapy may
be required.99

At present, routine treatment of asymptomatic disease is not recommended (Fig.


3).58,100 Previous studies have suggested a high spontaneous resolution of bacterial
vaginosis.58 Despite concern of the risk of PID in untreated patients with bacterial
vaginosis, most infections are uncomplicated. Thus, unless a vaginal surgical
procedure is contemplated or the patient is pregnant,58,100 treatment of the
asymptomatic patient is not indicated. Pregnant patients and those undergoing vaginal
surgical procedures should be screened and appropriately treated prior to delivery or
the vaginal surgical procedure. Recommendations may change as our understanding
of this infection grows.

Trichomonas vaginalis. 1000. Organism with flagella seen on normal saline wet
preparation.

TRICHOMONAS VAGINITIS

Pathology

The incidence of trichomonas infection has been declining for the past 20 years, and
currently this accounts for only 10% to 25% of vaginal infections, depending on the
population studied.33,101,102 Sexually transmitted disease clinics report a higher incidence
of this infection. Risk factors for trichomonas infection include lower socioeconomic
status and having a greater number of sexual partners. Cigarette smoking as well as
African-American race have also been linked to trichomonas infection, even after
adjustment for income and level of education. In addition, women who stop smoking
are no more likely to be infected than nonsmokers, suggesting that smoking increases
trichomonas colonization through a physiologic effect on the vaginal
environment.101 The organism has been found to coexist with a number of other
infections, including gonococcal infection and bacterial vaginosis.11,103 Concurrent
bacterial vaginosis and trichomoniasis were found in 22% of pregnant adolescents in
one study.104 Trichomonads generate hydrogen, which combines with oxygen,
removing oxygen from the vaginal ecosystem.105 This may facilitate growth of other
anaerobic bacteria and predispose the person to bacterial vaginosis, although this has
not been confirmed.

The organism, slightly larger than a leukocyte, is a motile protozoan with four
flagella. Its vigorous mechanical motion is felt to be cytotoxic, and causes a variable
amount of vaginal mucosal erythema.106The characteristic strawberry cervix, caused
by intraepithelial hemorrhages, is seen in only 5% to 10% of patients.2,101 Virulence is
highly variable among the different strains, and 25% to 50% of women who are
culture positive for Trichomonas vaginalis are asymptomatic. Trichomonas
vaginalis can be recovered from the urine in up to 30% of male sexual contacts of
women with the infection; however, less than 20% of men with trichomonas in the
urine are symptomatic.104 Trichomonas infection is a sexually transmitted disease.
However, sexual infection may not be the sole mode of transmission. The organism
has been shown to survive in hot tubs, tap water, and chlorinated swimming pools.
These modes of spread are uncommon, but should be considered when recurrent
infection is a problem.107

Trichomonas infection is also associated with complications, including abnormal Pap


tests 40 and PID.108Abnormal Pap tests are more frequent, however, in patients with
bacterial vaginosis.69Trichomonas acts as a vector for the development of PID, with
the motility of the organism facilitating movement of microorganisms from the vagina
to the fallopian tubes.108 Premature rupture of membranes in pregnancy and neonatal
respiratory tract infection have also been associated with this organism.78,109 It is
difficult to determine the independent impact of T. vaginalis in adverse pregnancy
outcomes because of the association of trichomonas infection with smoking and lower
socioeconomic status.110

Clinical Presentation

Women with trichomonas vaginitis generally present with a malodorous, profuse,


frothy, watery, green discharge.11 Pruritus is a more frequent symptom than in patients
with bacterial vaginosis, but less common than in those with yeast vaginitis. Some
patients report very little in the way of symptoms.105 The vulvar and intertriginous
groin areas are spared, and the vaginal rugations are typically only mildly inflamed.

Diagnosis

Copious watery discharge is usually evident, which may be gray to green in color. The
discharge has a normal to very alkaline pH, usually 6 to 7, and many white blood cells
are often present.105 The normal saline wet mount, which reveals the organism 60% to
75% of the time, is the most cost-effective method of diagnosis.111 Although a Pap test
can suggest trichomonas infection, the test produces significant numbers of false-
positive and false-negative results. Trichomonas infection found by Pap smear should
be confirmed by wet mount or culture prior to treatment.3,40,111

The most sensitive diagnostic modality is a trichomonas culture, which is relatively


inexpensive.111,112Despite the low cost, many laboratories do not culture trichomonas.
This requires a special liquid medium (Diamond's) that allows growth of the organism
so that it can be subsequently visualized on wet mount. The In Pouch TV culture
system has been found to perform as reliably as Diamond's medium in recoveringT.
vaginalis isolates and may be an alternative to traditional culture techniques.74 With
the increasing incidence of metronidazole resistance,112 cultures may become more
standard practice.2 Several other methods are also being evaluated for the diagnosis of
trichomonas infection, including a polymerase chain reactionbased test,26,113 a direct
fluorescent antibody test, and an enzyme immunoassay.110 Sensitivities and
specificities of these tests, however, are not clearly established.108 Fluorescent
antibody tests are commercially available; however, the sensitivity of these tests is
low, and use of a fluorescent microscope is needed.

Treatment

Options for the treatment of trichomonas infection include metronidazole at 500 mg


orally twice a day for 7 days, 250 mg orally three times a day for 7 days, or 2 g orally
in a single dose. Any of these regimens results in a cure rate of 86% to 97% when the
sexual partner is treated concurrently, as recommended by the Centers for Disease
Control and Prevention.107,114117 The FDA has approved a 375-mg dose orally twice a
day for 7 days, as serum levels are similar to the 250-mg regimen three times a
day.96 However, the single-dose regimen is preferable for maximizing patient
compliance. Topical intravaginal metronidazole is not effective treatment for
trichomonas infection, as adequate levels of medication are not achieved in the
urethra or perivaginal glands.96

Recommendations for treatment in pregnancy are changing with the evident safety of
metronidazole in pregnancy,97 and the association of trichomonas infection with
adverse pregnancy outcomes. A regimen of 2 g of metronidazole in a single dose is
the treatment of choice for trichomonas infection in women in all trimesters of
pregnancy.96 Patients allergic to metronidazole should be desensitized.96 Clotrimazole,
100 mg intravaginally at bedtime for 5 to 10 days, has been used for symptomatic
relief, although this is not an FDA-approved indication for this medication.118 If
invasive technologies such as chorionic villous sampling or cerclage placement are
necessary in pregnancy, preprocedure prophylaxis in infected patients is indicated to
avoid procedure-related infection and complications.105
Resistant trichomonas strains have been reported. Before assuming that an infection is
resistant to metronidazole, it is important to ascertain the compliance of the patient
and her partner. The concurrent use of medications such as phenobarbitol and
phenytoin should be determined, as they can interfere with the action of
metronidazole and decrease its effectiveness. When true resistance is present, it is
generally a relative resistance, and higher doses of metronidazole, such as 750 mg
four times a day for 6 to 10 days or longer, usually result in a cure.110 Very highly
resistant trichomonas infection has been reported, but this is rare (1 in 2,000 to 3,000
cases).110 A combination of vaginal metronidazole by suppository, 500 mg a day for 5
days, concurrently with oral metronidazole therapy has been effective in these
cases.105,110Intravenous metronidazole regimens (500 mg every 68 h for 3 d) have also
been used for resistant cases.119 Daily doses greater than 3 g of metronidazole can
cause neurologic injury, and this should be considered when choosing a regimen.
Intravaginal nonoxynol-9 relieves symptoms of trichomonas vaginitis, but seldom
results in cure.110,119 Tinidazole, available in Canada and Europe, can achieve cure in
some cases, because cross-resistance with metronidazole is incomplete.110

SUMMARY

Vaginitis is a common medical problem in women that can be associated with


significant morbidity and complications. Strain typing of Candida permits a better
understanding of recurrent yeast infection that can aid in its treatment. More
standardized methods of diagnosis in bacterial vaginosis such as Nugent's criteria are
important in further assessing potential complications associated with bacterial
vaginosis including premature rupture of membranes, chorioamnionitis, preterm
delivery, postpartum endometritis, wound infection, and PID. Patients who are
pregnant and in need of an invasive procedure or any woman undergoing pelvic
surgery should be screened and treated prior to the procedure. Topical treatments for
bacterial vaginosis including clindamycin cream and metronidazole gel have
decreased the need for systemic treatment, but should be used with caution in
pregnant patients, and cure should be documented. Trichomonas infection is also
associated with upper genital tract infection and an increasing incidence of
metronidazole resistance, which should be considered in patients with recurrent
infection. Research in recent years has provided improved diagnostic tools as well as
treatment modalities for all forms of vaginitis.

14. Ebm yg relevan yg brhubungn dg patofisiologi perdarahan post partum


15. Vasukularisasi dan anatomi reproduksi dan jalan lahir
16. Definisi dan etio demam post partum
17. DD demam post partum
18. Tanda kegawatan perdarahan post partum

You might also like